Cardiac surgery and interventional cardiology Flashcards

1
Q
A
How well did you know this?
1
Not at all
2
3
4
5
Perfectly
2
Q

An adult patient undergoing cardiac surgery exhibits excessive bleeding following
cardiopulmonary bypass. A thromboelastogram performed on their blood is shown
below. The most likely cause of the bleeding is

(ROTEM with low Extem A10 and normal Fibtem A10)

a) Platelets
b)Fibrinogen
c) FFP
d) TXA

A

Plateltes

Fibrinogen if low Fibtem
TXA if curves tail off early
FFP if MCF low

How well did you know this?
1
Not at all
2
3
4
5
Perfectly
3
Q

A transjugular intrahepatic portosystemic shunt procedure is contraindicated in
patients with:

a) Hepatorenal syndrome
b) Refractory ascites
c) Severe TR
d) Variceal bleeding
e) Budd chiari

A

c) Severe tricuspid regurgitation (TR)

Severe TR can lead to increased right atrial pressure, which may impede the proper function of the TIPS and worsen outcomes.

Contraindications:
Severe Hepatic encephalopathy
Severe Pulmonary Htn
Severe TR
Multiple Hepatic Cysts
Coagulopathy (relative contraindication)

How well did you know this?
1
Not at all
2
3
4
5
Perfectly
4
Q

The image below shows the arterial pressure (red, upper line) and balloon pressure
(blue, lower line) from an intra-aortic balloon pump set at 1:2 augmentation. The
point of the waveform indicated by the large green arrow is called:

a) Assisted end diastolic
b) Assisted systolic
c) Unassisted end diastolic
d) Assisted systolic

A

Assisted end diastolic

How well did you know this?
1
Not at all
2
3
4
5
Perfectly
5
Q

Organ procurement after circulatory death is generally stood down if the time from
cessation of cardiorespiratory support to circulatory death extends beyond:

a) 60min
b) 90min
c) 120min

A

90 mins

30mins
Liver
Pancreas
Heart

60mins
Kidneys

90mins
Lungs

Page 35 ANZICS statement 2.4.3 Warm ischemia time

Donate life

How well did you know this?
1
Not at all
2
3
4
5
Perfectly
6
Q

The bipolar leads of a 12-lead electrocardiogram are:

a) All
b) V1-V6
c) aVL, aVR, aVF
d) I, II, III
e) None

A

D) I, II, III

3-electrode system
- Uses 3 electrodes (RA, LA and LL)
- Monitor displays the bipolar leads (I, II and III)
Life in the Fast Lane

How well did you know this?
1
Not at all
2
3
4
5
Perfectly
7
Q

The time for reversal of therapeutic dabigatran after administration of
idarucizumab 5 g is:

a) 5 mins
b) 15 mins
c) 30 mins
d) 60 mins
e) 120 mins

A

5 mins
- Essentially one circulation time

Intravenously administer the dose of 5 g (2 vials, each contains 2.5 g) as
o Two consecutive infusions or
o Bolus injection by injecting both vials consecutively one after another via syringe

Idarucizumab was administered as one 5 g intravenous infusion over five minutes

Among the 90 patients with available data, the median maximum reversal of the pharmacodynamic anticoagulant effect of dabigatran as measured by ECT or dTT in the first 4 hours after administration of 5 g idarucizumab was 100%, with most patients (>89%) achieving complete reversal. Reversal of the pharmacodynamics effects was evident immediately after administration.

FDA Product Guide

See blue book article

How well did you know this?
1
Not at all
2
3
4
5
Perfectly
8
Q

Interference with pacemaker function can result from all of the following EXCEPT:

a) RF ablation
b) High volume ventilation
c) Peripheral nerve stimulator
d) CT
e) Diathermy

A

d) CT

British Heart Rhythm Societies guidelines

How well did you know this?
1
Not at all
2
3
4
5
Perfectly
9
Q

When auscultating the heart the Valsalva manoeuvre will increase the murmur
intensity of:

a) AS
b) MS
c) MR
d) MVP
e) VSD

A

Mitral valve prolapse

Valsalva increases the strength of murmurs due to hypertrophic obstructive cardiomyopathy and mitral valve prolapse. It decreases the intensity of aortic stenosis, mitral stenosis, aortic regurgitation, mitral regurgitation, and ventricular septal defects.
OPPOSITE IS TRUE FOR SQUATTING (Increases preload)

How well did you know this?
1
Not at all
2
3
4
5
Perfectly
10
Q

A patient with a perioperative troponin rise above normal, chest pain, left ventricular
anterior regional wall motion abnormality, and atheroma without thrombus
occluding 70% of the left anterior descending coronary artery has had a/an

NSTEMI
STEMI
Unstable angina
Acute myocardial injury
Chronic myocardial injury
Type 1 MI
Type 2 MI

A

NSTEMI

MINS: MI/ischemic myocardial injury that doesn’t fulfill MI defn

MI: Myocardial injury with rise/fall cTn above 99th percentile of upper ref limit within 30 days post op plus at least one of:
Ischemic symptoms
New ischemic ECG changes
New path Q waves on ECG
Imaging evidence of myocardial ischemia
Angiographic/autopsy evidence of coronary thrombus

How well did you know this?
1
Not at all
2
3
4
5
Perfectly
11
Q

NP A medication that has NOT been associated with arrhythmogenic potential in patients with Brugada syndrome is:

a) Propofol
b) Thiopentone
c) Amiodarone
d) Ketamine

A

MAYANK B Thiopentone

BJA article 2018

Propofol infusions have been associated with a brugada like ECG.

How well did you know this?
1
Not at all
2
3
4
5
Perfectly
12
Q

The abnormalities seen in the electrocardiogram below are consistent with:

a) Hypercalcaemia
b) Hypermagnasaemia
c) Hyperphosphataemia
d) Hypokalaemia
e) Hyperkalaemia

A

NAOMI

ECG features of hypokalemia:

Increased P wave amplitude
Prolongation of PR interval
Widespread ST depression and T wave flattening/inversion
Prominent U waves (best seen in the precordial leads V2-V3)
Apparent long QT interval due to fusion of T and U waves (= long QU interval)

How well did you know this?
1
Not at all
2
3
4
5
Perfectly
13
Q

You are anaesthetising an 18-year-old who has a Fontan circulation for exploratory laparotomy. They are intubated and ventilated with a ventilator that has been brought from the Intensive Care Unit. Their current arterial oxygen saturation is 70%. To improve oxygenation, you should INCREASE the:

a) Increase PIP
b) Increase PEEP
c) Increase inspiratory time
d) Increase expiratory time

A

D) increase expiratory time

Reworded repeat, but prev options don’t directly align with these

Answer from then

Patients who have undergone the Fontan procedure depend on blood flow through the pulmonary circulation without the assistance of the right ventricle. The difference between central venous pressure and systemic ventricular end-diastolic pressure (termed the “transpulmonary gradient”) is the primary force promoting pulmonary blood flow and, more importantly, cardiac output.

Circulation in the Fontan patient is promoted by low pulmonary vascular resistance. Positive-pressure ventilation with increased tidal volumes, as described above, can result in excessive intrathoracic pressures, leading to decreased venous return to the heart and increased pulmonary vascular resistance.
In periods of low oxygen saturation, 100% inspiratory oxygen is appropriate.
The addition of PEEP will increase intrathoracic pressure, reducing venous return.
Trendelenberg positioning would increase CVP and therefore bloodflow through pulmonary circulation.

BJA: fontan circulation:
For relatively short procedures, Fontan patients are probably better off breathing spontaneously, as long as severe hypercarbia is avoided. For major surgery, or when prolonged anaesthesia is required, control of ventilation and active prevention of atelectasis is usually advisable. Potential disadvantages of mechanical ventilation in Fontan patients relate to the inevitable increase in mean intrathoracic pressure. This causes decreased venous return, decreased pulmonary blood flow, and hence, decreased cardiac output. Low respiratory rates, short inspiratory times, low PEEP, and tidal volumes of 5–6 ml kg−1 usually allow adequate pulmonary blood flow, normocarbia, and a low PVR. Hyperventilation tends to impair pulmonary blood flow, despite the induced respiratory alkalosis, because of the increased mean intrathoracic pressure.

https://academic.oup.com/bjaed/article/8/1/26/277637

How well did you know this?
1
Not at all
2
3
4
5
Perfectly
14
Q

You are called to assist with a patient in the intensive care unit who has had cardiac surgery three days ago and is now in cardiac arrest. External cardiac massage should aim for a systolic blood pressure of

a. 40
b. 60
c. 80
d. 100
e. 120

A

REPEAT

b. 60

BJA Article - ​Management of cardiac arrest following cardiac surgery - BJA Education

In the CICU, the effectiveness of ECC is confirmed by monitoring the arterial pressure trace with a target compression rate and depth to achieve a systolic impulse of > 60 mm Hg to maintain a mean perfusion pressure, preventing ventricular distension, LV wall stress, and ischaemia.

How well did you know this?
1
Not at all
2
3
4
5
Perfectly
15
Q

A 65-year-old man is undergoing coronary artery bypass grafting. Immediately upon commencing cardiopulmonary bypass and prior to administering cardioplegia, the aortic line blood appears the same colour as the blood in the venous cannulae, and the low venous saturation alarm is activated on the bypass machine. The most appropriate management at this point is to:

a) Attach another oxygen tubing to oxygenator
b) Increase the oxygen mix with air:oxygen blender
c) Ventilate and wean bypass

alternative remembered answers:
- Ventillate with 100% and continue
- Give O2 via side line
- Wean from bypass and ventilate lungs
- Inc blender FiO2

A

*Failure of oxygenation

I think wean bypass and ventilate - thoughts?

How well did you know this?
1
Not at all
2
3
4
5
Perfectly
16
Q

During rewarming on cardiopulmonary bypass, the most reliable surrogate for cerebral temperature measurement is:

A) Nasopharynx
B) Oxygenator arterial outlet
C) Oxygenator venous inflow
D) Bladder temp
E) PA Cath

A

A) Nasopharyngeal

Proximity to brain
Consistent correlation with core and brain temperature changes
Accessibility

Oxygenator blood temp represents temp of blood leaving circuit and doesnt reflect blood perfusing the brain

Clinical Techniques in Cardiovascular and Thoracic Surgery: This textbook discusses the monitoring of cerebral temperature during CPB and often cites nasopharyngeal temperature as a standard method due to its proximity to the brain.

Reference: Sabik, Joseph F., et al. “Temperature management and monitoring during cardiopulmonary bypass.” In: Clinical Techniques in Cardiovascular and Thoracic Surgery, edited by Little Brown and Company, 1998.
Perfusion: This journal article discusses various techniques for monitoring cerebral temperature during CPB, emphasizing the use of nasopharyngeal temperature probes.

Reference: Zollinger, Andreas, et al. “Temperature Management and Monitoring During Cardiopulmonary Bypass.” Perfusion, vol. 18, no. 1, 2003, pp. 3-9. doi:10.1191/0267659103pf582oa.

How well did you know this?
1
Not at all
2
3
4
5
Perfectly
17
Q

The image below is from the transoesophageal echocardiogram of an adult patient who is about to undergo cardiac surgery. The structure labelled with the arrow is the:

TOE image - four chamber, with arrow pointing to leaflet closest to septum

A) Anterior mitral leaflet
B) Posterior mitral leaflet
C) Tricuspid septal leaflet
D) Tricuspud anterior leaflet
E) Tricuspid posterior leaflet

A

A) Anterior mitral leaflet

How well did you know this?
1
Not at all
2
3
4
5
Perfectly
18
Q

A 45-year-old received a heart transplant one month ago. They develop a new supraventricular tachyarrhythmia without hypotension during gastroscopy. The most appropriate therapy is:

a) Adenosine
b) Amiodarone
c) Esmolol
d) Verapamil
e) Digoxin

A

REPEAT

d) Esmolol

Management of Arrhythmias After Heart Transplant
https://www.ahajournals.org/doi/10.1161/CIRCEP.120.007954

In asymptomatic patients, additional cardiac monitoring such as 24-Holter or an event monitor can be useful to assess the SVT burden, and a trial of atrioventricular nodal blockers (β-blockers preferably) can be attempted with caution in view of potential risk of bradycardia. Calcium channel blockers such as diltiazem and verapamil are contraindicated in patients taking immunosuppression such as tacrolimus and cyclosporine as it can impair the metabolism CYP3A, which increases the levels of these drugs potentially causing renal toxicity.

The use of adenosine in the management of SVT has remained a subject of controversy for over a quarter century. In the past, adenosine was contraindicated in patients post-OHT due to its supersensitivity and presumed risk of prolonged atrioventricular block.

Thus, based on the aforementioned data, in patients with OHT, adenosine is feasible and safe at reduced doses (starting at 1.5 mg for patients ≥60 kg) as long as patients are closely monitored, with dose escalation as needed. Furthermore, the 2010 American Heart Association guidelines on advanced cardiovascular life support also recommended lowering the initial dose of adenosine to 3 mg for the acute management of SVT in patients with OHT.

How well did you know this?
1
Not at all
2
3
4
5
Perfectly
19
Q

An 85-year-old is scheduled for open reduction and internal fixation of a fractured neck of femur today. They have no significant past medical history. Preoperative review including physical examination, full blood count, electrolyte profile and electrocardiogram performed yesterday were normal. In the anaesthetic bay, the monitor shows the patient to be in atrial fibrillation with a ventricular rate of 110 to 145 beats per minute. The blood pressure is 130/80 mmHg. The best initial treatment for the atrial fibrillation is:

A) Amiodarone
B) Metoprolol
C) Digoxin
D) Induce then cardiovert
E) Calcium Channel Blocker

A

B) Metoprolol

Guidelines from the American College of Cardiology/American Heart Association (ACC/AHA) and the European Society of Cardiology (ESC) recommend beta-blockers as a first-line therapy for rate control in atrial fibrillation.
Reference: January CT, Wann LS, Calkins H, et al. 2019 AHA/ACC/HRS focused update of the 2014 AHA/ACC/HRS guideline for the management of patients with atrial fibrillation: a report of the American College of Cardiology/American Heart Association Task Force on Clinical Practice Guidelines and the Heart Rhythm Society. Circulation. 2019;140(2)

How well did you know this?
1
Not at all
2
3
4
5
Perfectly
20
Q

NP A 65-year-old presents with an acute dissection of their thoracic aorta. Their blood pressure is 150/90 mmHg. The best medication to reduce the blood pressure is:

a) Esmolol
b) SNP
c) GTN
d) Hydralazine

A

A) esmolol

They get anti impulse therapy which usually starts off with beta blockade before alpha blockade.

Up to date: Patients often present with severe hypertension and are initially stabilized with fast-acting, intravenous beta blockers (eg, esmolol or labetalol) or calcium channel blockers. Anti-impulse therapy lowers blood pressure

How well did you know this?
1
Not at all
2
3
4
5
Perfectly
21
Q

A 30-year-old athlete undergoing a knee arthroscopy under general anaesthesia
develops intraoperative tachycardia. A 12-lead electrocardiogram is obtained and
shown below. The most likely diagnosis is:

a) AF
b) Flutter
c) AVNRT
d) Multifocal atrial tachycardia

A

AT

Repeat

Delta waves present, therefore WPW = AVRT

WPW + delta wave = AVRT → anatomical re-entry circuit (Bundle of Kent)
AVNRT is a functional re-entry circuit within the AV node
ECG features of AVNRT
● Regular tachycardia ~140-280 bpm
● Narrow QRS complexes (< 120ms) unless there is co-existing bundle branch block, accessory pathway, or rate-related aberrant conduction
● P waves if visible exhibit retrograde conduction with P-wave inversion in leads II, III, aVF. They may be buried within, visible after, or very rarely visible before the QRS complex
https://litfl.com/supraventricular-tachycardia-svt-ecg-library/

How well did you know this?
1
Not at all
2
3
4
5
Perfectly
22
Q

A third heart sound at the apex may be heard in:

a) Healthy people aged less than 40
b) Mitral prolapse
c) HTN

23.1 OPTIONS:

a) pulmonary stenosis
b) pulmonary hypertension
c) pericarditis
d) pregnancy

A

AT

Repeat

Can occur in healthy young people

The third heart sound is mainly created by the early-diastolic rapid distension of the left ventricle that accompanies rapid ventricular filling and abrupt deceleration of the atrioventricular blood flow

S3 may be normal in people under 40 years of age and some trained athletes but should disappear before middle age. Re-emergence of this sound late in life is abnormal[5] and may indicate serious problems such as heart failure.

‘Sounds like Ken-tu-cky’

How well did you know this?
1
Not at all
2
3
4
5
Perfectly
23
Q

The 12-lead electrocardiogram shown is most consistent with acute total occlusion
of the:

a) LAD
b) PDA
c) OM
d) RCA

A

VICTORIA

Am I missing something? I can’t see total occlusion of anything here.

Wellens syndrome- Lad

How well did you know this?
1
Not at all
2
3
4
5
Perfectly
24
Q

A 55-year-old with no past history of ischaemic heart disease is three days post-total hip replacement surgery. They have an episode of chest pain at rest with features typical of angina that lasts 30 minutes before fully resolving. There are no electrocardiogram changes and no troponin rise. The diagnosis is

a. No diagnosis made
b. Unstable angina
c. STEMI
d. NSTEMI
e. MINS

A

REPEAT
b. Unstable angina

UTD:

Unstable angina (UA) and acute non-ST elevation myocardial infarction (NSTEMI) differ primarily in whether the ischemia is severe enough to cause sufficient myocardial damage to release detectable quantities of a marker of myocardial injury (troponins):

●UA is considered to be present in patients with ischemic symptoms suggestive of an ACS and no elevation in troponins, with or without electrocardiogram changes indicative of ischemia (eg, ST segment depression or transient elevation or new T wave inversion).

●NSTEMI is considered to be present in patients having the same manifestations as those in UA, but in whom an elevation in troponins is present.

MINS: Myocardial injury after non-cardiac surgery (up to 30 days post-op):
1. Elevated postop troponin
2. Resulting from myocardial ischaemia (i.e. no evidence of a non-ischaemic aetiology), not requiring an ischaemic feature (i.e. no chest pain, no ECG change)

VISION studies (Vascular Events in Noncardiac Surgery Patients Cohort Evaluation) demonstrated that severity of MINS strongly associated with 30-day mortality after NCS.

hs-cTnT
<20ng/L ~ 0.5% 30 day mortality
20-64ng/L ~3% 30 day mortality
65-999 ng/L ~9% 30 day mortality
>1000ng/L ~30% 30 day mortality

Whilst VISION trial identified MINS in at risk patients, the question now becomes what interventions are available to prevent this complication?

How well did you know this?
1
Not at all
2
3
4
5
Perfectly
25
Q

The QRS axis of the attached electrocardiograph is closest to:

a) -90
b) -45
c) +45
d) +90

A

VICTORIA

ECG sent to JJ

B

https://litfl.com/super-axis-man-sam/

How well did you know this?
1
Not at all
2
3
4
5
Perfectly
26
Q

Kate
In cardiac surgery a low-normal central venous pressure and a low blood pressure with a hyperdynamic heart is suggestive of:

a) Vasoplegia
b) Hypovolaemia

A

A) vasoplegia
Vasoplegia is characterized by a normal or augmented cardiac output with low systemic vascular resistance (SVR) causing organ hypoperfusion. The exact definition has varied but typically is considered when shock occurs within 24 h of CPB in the setting of a cardiac index (CI) is greater than 2.2 L/kg/m2 and SVR less than 800 dyne s/cm5

How well did you know this?
1
Not at all
2
3
4
5
Perfectly
27
Q

Kate
In an adult weighing 70 kg, a bedside assessment of haemodynamic status shows a left ventricular end-diastolic diameter of 2.4 cm. This finding suggests:

a) Hypovolaemia
b) Normal
c) Hypervolaemia

A

? Normal
Can only find absolute numbers or according to BSA not weight per se

Image
https://thoracickey.com/cardiac-chambers/

PSAX End diastolic AREA:
Hypovolemia <8cm2
Normal 8-14cm 2
Hypervolemia > 14cm2
IVSd and IVSs – Interventricular septal end diastole and end systole. The normal range is 0.6-1.1 cm.
LVIDd and LVIDs – Left ventricular internal diameter end diastole and end systole. The normal range for LVIDd is 3.5-5.6 cm, and the normal range for LVIDs is 2.0-4.0 cm.
LVPWd and LVPWs – Left ventricular posterior wall end diastole and end systole. The normal range is 0.6-1.1 cm.
RVDd – Right ventricular end diastole. The normal range is 0.7-2.3 cm.
Ao Root Diam – Aortic root diameter. The normal range is 2.0-4.0 cm.
LA Diameter – Left atrium diameter. The normal range is 2.0-4.0 cm.
The IVSd and IVPWd measurements are used to determine left ventricular hypertrophy, which is the thickening of the muscle of the left ventricle. LV hypertrophy is a marker for heart disease. In general, a measurement of 1.1-1.3 cm indicates mild hypertrophy, 1.4-1.6 cm indicates moderate hypertrophy, and 1.7 cm or more indicates severe hypertrophy.

Hypovolaemia
Normal for end diastole is 3.5 to 5.6cm

How well did you know this?
1
Not at all
2
3
4
5
Perfectly
28
Q

22.2 The most likely diagnosis for the following electrocardiograph is
(comment that this was like a 2015A repeat - ECG below is from that paper + 2022 recalled options)

a. AF with BBB
b. sinus tachy with BBB
c. ventricular tachycardia
d. torsades

A

b. sinus tachy with BBB

The most correct answer would be Trifasicular block:
RBBB with LAD (RBBB with left anterior hemiblock) and 1st degree heart block

Barash 8E 2017:
The term bifascicular block often refers to block in the right bundle and one of the two major fascicles of the left bundle. RBBB with left anterior hemiblock is present when the ECG shows an RBBB with a left axis deviation (usually greater than −60 degrees) in the absence of an inferior myocardial infarction. Complete RBBB with right axis deviation (greater than 90 degrees) is indicative of RBBB and left posterior hemiblock in the absence of a lateral myocardial infarction or evidence of right-sided heart failure. The term trifascicular block is used to describe first-degree AV block in the presence of bifascicular block.

Is it necessary to insert a temporary pacemaker before general anesthesia for an asymptomatic patient with bifascicular or trifascicular block?
The risk for progression to complete heart block in asymptomatic patients with bifascicular block is low. Further, no clinical characteristics have been identified that accurately predict the risk of development of complete heart block. Therefore, routine PPM implantation in patients with asymptomatic bifascicular block is not recommended. Observations made in the perioperative period have suggested that development of complete heart block during general anesthesia is also rare; therefore, it is generally not recommended that patients undergo temporary pacemaker insertion before general anesthesia. However, it is advisable to have an external pacemaker available in the operating room.

How well did you know this?
1
Not at all
2
3
4
5
Perfectly
29
Q

22.1 A four-year-old boy is in refractory ventricular fibrillation. The recommended dose of amiodarone is

A

80mg

Age + 4 x 2-> 4 + 4 x 2 =16kg
5 x 16mg =80mg

How well did you know this?
1
Not at all
2
3
4
5
Perfectly
30
Q

22.2 An absolute contraindication to transoesophageal echocardiography is
A. Dysphagia
B. GORD
C. Oesophageal stricture
D. oesophageal webbing
E. oesophageal varices

A

C. Oesophageal stricture

https://www.asecho.org/wp-content/uploads/2014/05/2013_Performing-Comprehensive-TEE.pdf

How well did you know this?
1
Not at all
2
3
4
5
Perfectly
31
Q

22.2 A four-year-old boy is in refractory ventricular fibrillation. The recommended dose of amiodarone is

a) 40mg
b) 80mg
c) 120mg
d) 160mg

A

80mg

Age + 4 x 2-> 4 + 4 x 2 =16kg
5 x 16mg =80mg

16kg x 5mg/kg = 80mg

How well did you know this?
1
Not at all
2
3
4
5
Perfectly
32
Q

20.1 In a Blalock–Taussig shunt, blood passes to the pulmonary artery via the

a. Aorta
b. Subclavian artery
c. IVC
d. SVC
e. Left atrium

A

B

How well did you know this?
1
Not at all
2
3
4
5
Perfectly
33
Q

23.1 Application of a pacemaker magnet to a dual-chamber implanted pacemaker would be expected to convert the operating mode to

a. AOO
b. VOO
c. DOO
d. AAI

A

c) DOO

The pacing mode will be DOO when the programmed pacing mode is a dual chamber mode or an MVP mode (AAIR<=>DDDR, AAI<=>DDD), VOO when the programmed pacing mode is a single chamber ventricular mode, and AOO when the programmed pacing mode is a single chamber atrial mode.

How well did you know this?
1
Not at all
2
3
4
5
Perfectly
34
Q

21.2 The image below on the left shows a normal central venous pressure (CVP) trace. The CVP
trace in the image below on the right is most consistent with

a) AF
b) MR
c) AR
d) TR
e) Pericardial constriction

A

TR

How well did you know this?
1
Not at all
2
3
4
5
Perfectly
35
Q

20.1 ECG calibration, 10mm on Y axis is equal to:

a. 0.2 sec
b. 0.4sec
c. 1sec
d. 0.1mV
e. 1mV

A

b) 1mV

How well did you know this?
1
Not at all
2
3
4
5
Perfectly
36
Q

22.1 A 68-year-old woman presents with a loud systolic murmur in the anaesthesia room before total
hip joint arthroplasty. A transthoracic echocardiogram is performed (image provided) and shows

a. AS
b. LVOT
c. MR

A

MR

How well did you know this?
1
Not at all
2
3
4
5
Perfectly
37
Q

21.2 Of the following, the lifestyle modification that is least effective in reducing essential
hypertension is

a) Stopping caffeine
b) Low salt diet
c) High potassium diet
d) Exercise
e) Alcohol cessation

A

a) Stopping caffeine

Eat a well-balanced diet that’s low in salt
Limit alcohol
Enjoy regular physical activity
Manage stress
Maintain a healthy weight
Quit smoking

Foods that are rich in potassium are important in managing high blood pressure (HBP or hypertension) because potassium lessens the effects of sodium. The more potassium you eat, the more sodium you lose through urine. Potassium also helps to ease tension in your blood vessel walls, which helps further lower blood pressure.

Source AHA

How well did you know this?
1
Not at all
2
3
4
5
Perfectly
38
Q

21.1 The management of a patient who has experienced a cardiac arrest within 10 days of cardiac surgery
should NOT routinely include

a. Atropine 3mg
b. adrenaline 1mg boluses
c. 3 stacked shocks
d. amiodarone 300
e. 1L fluid

A

b. adrenaline 1mg boluses

The risk of administering adrenaline in conventional doses is with profound hypertension, bleeding, or tearing of vessel anastomoses on return of spontaneous circulation (ROSC), which can precipitate catastrophic harm or further cardiac arrest.
Adrenaline remains a useful drug in peri-arrest situations in smaller doses.

How well did you know this?
1
Not at all
2
3
4
5
Perfectly
39
Q

23.1 The following is a chest X-ray from a patient complaining of dyspnoea after thoracic surgery. The diagnosis is

(not the image from the exam)

A. Dextracardia
B. Cardiac hernation
C. LLL collapse
D. Tension Pneumohorax

A

B. Cardiac hernation

https://www.ahajournals.org/doi/10.1161/CIRCULATIONAHA.109.896829

How well did you know this?
1
Not at all
2
3
4
5
Perfectly
40
Q

20.2 A 56 year old patient presents with exertional syncope. The most likely diagnosis is

a) HOCM
b) Long QT
c) CCF
d) Myocardial ischaemia

A

HOCM if these remembered options are correct

Alternative is Aortic Stenosis which is more common than HOCM in this age group

How well did you know this?
1
Not at all
2
3
4
5
Perfectly
41
Q

20.2 Cardiovascular effects of hyperthyroidism include

a) Decreased CO
b) Increased PVR
c) Increased DBP
d) Decreased SVR

A

Decreased SVR

Hyperthyroidism:
increases HR
increases cardiac contractility.
increases LVEF
increases diastolic relaxation
increases CO

SVR decreases
>T3 induces systemic vasodilation.

How well did you know this?
1
Not at all
2
3
4
5
Perfectly
42
Q

20.2 According to National Audit Project (NAP) 5, the incidence of awareness during general anaesthesia for cardiac surgery is

a) 1:400
b) 1:800
c) 1:8000
d) 1: 12000
e) 1:20000

A

c) 1:8000

Awareness rates
GA with no muscle relaxant = 1:136,000
GA with muscle relaxation = 1/8,000
CTS 1/8,600
E-LSCS = 1/670
Overall 1:19,000

How well did you know this?
1
Not at all
2
3
4
5
Perfectly
43
Q

23.1 In order to minimise the risk of cardiac arrhythmia, surgical diathermy has been designed to operate with

A. High frequency
B. High amplitude
C. Low frequency
D. Low amplitude
E. Using EES

A

A. High frequency

How well did you know this?
1
Not at all
2
3
4
5
Perfectly
44
Q

22.2 Based on this ECG tracing, the mode in which this pacemaker is operating is

a) VAI with intermittent failure to capture
b) AAI with intermittent failure to sense
c) DDD
d) VVI with intermittent failure to capture
e) VVI with intermittent failure to sense

A

e) VVI with intermittent failure to sense

How well did you know this?
1
Not at all
2
3
4
5
Perfectly
45
Q

20.1 Cardiovascular effects of hyperthyroidism include

a) decreased diastolic relaxation
b) decreased SVR
c) decreased PVR
d) increased diastolic BP

A

Decreased SVR
- increased CO, increased SBP and decreased DBP with widened PP

Up to Date
Cardiovascular - Patients with hyperthyroidism have an increase in cardiac output, due both to increased peripheral oxygen needs and increased cardiac contractility. Heart rate is increased, pulse pressure is widened, and peripheral vascular resistance is decreased

How well did you know this?
1
Not at all
2
3
4
5
Perfectly
46
Q

22.2 A 45-year-old male received a heart transplant one month ago. He develops a new supraventricular tachyarrhythmia without hypotension during a gastroscopy. The most appropriate therapy is

a) Adenosine
b) Amiodarone
c) Digoxin
d) Esmolol
e) Verapamil

A

d) Esmolol

Management of Arrhythmias After Heart Transplant
https://www.ahajournals.org/doi/10.1161/CIRCEP.120.007954

In asymptomatic patients, additional cardiac monitoring such as 24-Holter or an event monitor can be useful to assess the SVT burden, and a trial of atrioventricular nodal blockers (β-blockers preferably) can be attempted with caution in view of potential risk of bradycardia. Calcium channel blockers such as diltiazem and verapamil are contraindicated in patients taking immunosuppression such as tacrolimus and cyclosporine as it can impair the metabolism CYP3A, which increases the levels of these drugs potentially causing renal toxicity.

The use of adenosine in the management of SVT has remained a subject of controversy for over a quarter century. In the past, adenosine was contraindicated in patients post-OHT due to its supersensitivity and presumed risk of prolonged atrioventricular block.

Thus, based on the aforementioned data, in patients with OHT, adenosine is feasible and safe at reduced doses (starting at 1.5 mg for patients ≥60 kg) as long as patients are closely monitored, with dose escalation as needed. Furthermore, the 2010 American Heart Association guidelines on advanced cardiovascular life support also recommended lowering the initial dose of adenosine to 3 mg for the acute management of SVT in patients with OHT.

How well did you know this?
1
Not at all
2
3
4
5
Perfectly
47
Q

22.1 A 74-year old man complains of chest pain. An electrocardiograph is performed and displayed here. The occluded coronary artery could be the

a) RCA or LCx
b) RCA
c) LAD

A

RCA or LCx

https://litfl.com/mi-localization-ecg-library/

How well did you know this?
1
Not at all
2
3
4
5
Perfectly
48
Q

22.1 A risk factor for the development of torsade de pointes is

a. hyperkalaemia
b. hypermagnasaemia
c. tachycardia
d. Female

A

d. Female

How well did you know this?
1
Not at all
2
3
4
5
Perfectly
49
Q

22.1 A drug which does NOT increase the defibrillation threshold in a patient with an implanted cardioverter defibrillator is

a. Amiodarone
b. Atropine
c. B-blocker
d. Flecainide
e. Sotalol

A

e. Sotalol

Drugs that INCREASE defibrillation threshold:
+ Amiodarone (Chronic)
+ Atropine
+ lignocaine
+ Diltiazem
+ Flecainide
+ Verapamil
+ Venlafaxine
+ Anaesthetic agents.

Drugs that DECREASE defibrillation threshold:
- Sotalol
- Amiodarone (acute)
- Nifekalant

Drugs with No Change in DFT
= B- blocker
= Disopyramide
= Procainamide
= Propafenone

https://www.ncbi.nlm.nih.gov/pmc/articles/PMC6304797/

How well did you know this?
1
Not at all
2
3
4
5
Perfectly
50
Q

23.1 Findings associated with massive pericardial tamponade include

a. Electrical alternans
b. Exaggerated collapsible IVC on ECHO during respiratory cycle
c. Pulses alternans
d. Kussmaul breathing

A

a) electrical alternans

Physical findings in Tamponade:
- A number of findings may be present on physical examination, depending upon the type and severity of cardiac tamponade
- None of the findings alone are highly sensitive or specific for the diagnosis.

Beck’s triad
1. Low arterial blood pressure
2. Dilated neck veins
3. Muffled heart sounds
- Are present in only a minority of cases of acute cardiac tamponade.

Diagnosis:
Clinical diagnosis is usually suspected based on the history and physical examination findings, which may include:
●Chest pain
●Syncope or presyncope
●Dyspnea and tachypnea
●Hypotension
●Tachycardia
●Peripheral edema
●Elevated jugular venous pressure
●Pulsus paradoxus

How well did you know this?
1
Not at all
2
3
4
5
Perfectly
51
Q

20.1 A patient with multiple co-morbidities has severe symptomatic aortic stenosis and is considered for an aortic valve replacement. Compared to an open surgical approach, a transcatheter aortic valve implantation (TAVI) has

a) Reduced vascular injury
b) Reduced mean valve gradient
c) Reduced paravalvular leak
d) Reduced complete heart block
e) Reduced reintervention

A

b) Reduced mean valve gradient

TAVI decreased:
- AKI
- AF
- Transfusion
- Mean prosthetic valve gradient

TAVI increased:
- Major vascular complications
- Permanent pacemaker implantation
- Paravalvular regurgitation
- Need for re-intervention

How well did you know this?
1
Not at all
2
3
4
5
Perfectly
52
Q

21.2 Of the following drugs, the least likely to cause pulmonary vasodilation when used at low
doses in patients with chronic pulmonary hypertension is

a) Dopamine
b) Dobutamine
c) Vasopressin
d) Milrinone

A

dopamine

  • least likely to cause pulmonary vasodilation (all the others do to my knowledge)
  • From UP TO DATE:
    > At low doses of 1 to 3 mcg/kg per min, dopamine acts primarily on dopamine-1 receptors to dilate the renal and mesenteric artery beds
    > At 3 to 10 mcg/kg per min (and perhaps also at lower doses), dopamine also stimulates beta-1 adrenergic receptors and increases cardiac output, predominantly by increasing stroke volume with variable effects on heart rate.
    > At medium-to-high doses, dopamine also stimulates alpha-adrenergic receptors, although a small study suggested that renal arterial vasodilation and improvement in cardiac output may persist as the dopamine dose is titrated up to 10 mcg/kg per min
    *clinically, the haemodynamic effects of dopamine demonstrate individual variability

Dobutamine (inodilator):
- selective β1-agonist that increases cardiac contractility and reduces pulmonary vascular and systemic vascular resistances

Vasopressin:
- vasopressin may have pulmonary vasodilatory effects in addition to a systemic vasoconstrictive effect

Milrinone (inodilator):
- the phosphodiesterase-3 inhibitors, milrinone and enxoimone, have positive inotropic effects combined with the capacity to reduce RV afterload (‘inodilators’) without significant chronotropic effect, but they can be associated with significant systemic hypotension

How well did you know this?
1
Not at all
2
3
4
5
Perfectly
53
Q

20.2 You are called to assist in the resuscitation of a 75-year-old female patient in the emergency department who is hypotensive and hypoxaemic in extremis. The image shown is of a focused transthoracic echocardiogram, parasternal short axis view. The most likely diagnosis is

a) Pulmonary embolism
b) Anterior MI
c) Cardiac tamponade
d) Pneumothorax

A

a) Pulmonary embolism

A bit about the RV in PE:

The right ventricle drapes around the LV. In response to an acute Pulmonary Embolus (PE) it first dilates. The RV can’t generate much force without training, sowhen the Pulmonary Vascular Resistance (PVR) first rises with a PE, thepulmonary arterypressures don’t actually rise substantially because the RV can’t generate largepressures.

Looking at the ventricle in short axis, the septum maybow towardstheLV which will form aD shape indiastole,producing a“volumeoverloaded right ventricle” appearance.

Only later whenthe RV has beentrainedwill it be able togenerate higher pressures. If the LV is D shaped insystole, this is a “pressureoverloaded right ventricle”.

Acute cor pulmonale with bothpressureANDvolumeoverload (D shape insystoleANDdiastole)is often absent.

How well did you know this?
1
Not at all
2
3
4
5
Perfectly
54
Q

20.1 You are inserting a pulmonary artery catheter in an intubated patient prior to cardiac surgery, and a significant amount of blood appears in the endotracheal tube. The most appropriate specific initial management is to:

A. Remove PAC and insert DLT
B. Wedge PAC and insert DLT
C. Wedge PAC and insert bronchial blocker
D. Withdraw PAC 2 cm and insert DLT
E. Withdraw PAC and insert bronchial blocker
F. Inflate balloon

A

D. Withdraw PAC 2 cm and insert DLT

LITFL: Pulmonary haemorrhage after PAOP measurement

a life threatening time-critical emergency
pulmonary artery rupture caused by inflation of the pulmonary artery catheter (PAC) balloon during ‘wedging’ (measurement of the pulmonary artery occlusion pressure)
some experts advise against measuring PAWP because of the risk of pulmonary artery rupture
0.2% risk,  30% mortality

RISK FACTORS

pulmonary hypertension
mitral valve disease
anticoagulants
age >60 years

MANAGEMENT

Goals

prevent further pulmonary haemorrhage
stop bleeding
resuscitate

Call for help

ICU consultant
anaesthetist/ OT
cardiothoracic surgeon
interventional radiology

Resuscitation

A
    may have to be emergently intubated if not already
B
    FiO2 1.0
    controlled ventilation
    if able to recognize which lung is haemorrhaging may be able to perform lung isolation (insert single lumen tube into unaffected side, exchange for a double lumen tube or use bronchial blocker with bronchoscopic assistance)
    apply PEEP to tamponade wound
C
    large bore IV cannulae, fluids, blood products, inotropes

Specific therapy

Lay the patient ruptured side down
withdraw pulmonary catheter 2-3 cm with balloon down then refloat PAC with balloon inflated to occlude pulmonary artery (to try to tamponade bleeding)
stop antiplatelet agents and anticoagulants
give reversal agents:
— protamine for heparin
— platelets for anti-platelet agents
give blood products as indicated by FBC, coags and clinical state
interventions
— angiogram or bronchoscopy to isolate pulmonary vessel involved
— if bleeding doesn’t settle will require lobectomy
How well did you know this?
1
Not at all
2
3
4
5
Perfectly
55
Q

21.2 A 30 year old athlete undergoing a knee arthroscopy under general anaesthesia becomes tachycardic intraoperatively. A 12-lead electrocardiogram (ECG) is obtained. The most likely diagnosis is

a) Atrial fibrillation
b) Atrial flutter
c) Sinus tachycardia
d) WPW

A

d) WPW
Type B pattern

LITFL:
ECG features of WPW in sinus rhythm
-> PR interval < 120ms
-> Delta wave: slurring slow rise of initial portion of the QRS
-> QRS prolongation > 110ms
-> Discordant ST-segment and T-wave changes (i.e. in the opposite direction to the major component of the QRS complex)
-> Pseudo-infarction pattern in up to 70% of patients — due to negatively deflected delta waves in inferior/anterior leads (“pseudo-Q waves”), or prominent R waves in V1-3 (mimicking posterior infarction

Can be left-sided (Type A) or right-sided (Type B), and ECG features will vary depending on this:

Left-sided AP:
produces a positive delta wave in all precordial leads, with R/S > 1 in V1.
(Dominant R Wave in V1)
Sometimes referred to as a type A WPW pattern

Right-sided AP:
produces a negative delta wave in leads V1 and V2.
Sometimes referred to as a type B WPW pattern

Tachyarrhythmias in WPW

There are only two main forms of tachyarrhythmias that occur in patients with WPW

  1. Atrial fibrillation or flutter.
    -> Due to direct conduction from atria to ventricles via an AP, bypassing the AV node
  2. Atrioventricular re-entry tachycardia (AVRT).
    -> Due to formation of a re-entry circuit involving the AP

Breakdown of Type A example:
- Sinus rhythm with a very short PR interval (< 120 ms)
- Broad QRS complexes with a slurred upstroke to the QRS complex — the delta wave
- Dominant R wave in V1 suggests a left-sided AP, and is sometimes referred to as “Type A” WPW
- Tall R waves and inverted T waves in V1-3 mimicking right ventricular hypertrophy (RVH) — these changes are due to WPW and do not indicate underlying RVH
- Negative delta wave in aVL simulating the Q waves of lateral infarction — this is referred to as the “pseudo-infarction” pattern

How well did you know this?
1
Not at all
2
3
4
5
Perfectly
56
Q

23.1 Of the following drugs, the LEAST suitable for managing atrial arrhythmias in a patient with a left ventricular assist device is

A. Metoprolol
B. Amiodarone
C. Digoxin
D. Diltiazem

A

d) diltiazem

Nondihydropyridine calcium channel blockers should be used cautiously in patients with HFrEF because of their negative inotropic effects, and the role of these agents in LVAD recipients remains unclear

https://www.ahajournals.org/doi/10.1161/CIR.0000000000000673
Should also avoid sotolol

How well did you know this?
1
Not at all
2
3
4
5
Perfectly
57
Q

20.2 A 55 year old man with no past history of ischaemic heart disease is 3 days post total hip replacement surgery. He has an episode of chest pain that sounds ischaemic, began at rest and lasts thirty minutes before resolving fully. There are no ECG changes nor troponin rise. The diagnosis is

a. No diagnosis made
b. Unstable angina
c. STEMI
d. NSTEMI
e. MINS

A

b. Unstable angina

Not a Repeat, no Tropnin rise in this question making the answer unstable angina as opposed to NSTEMI

UTD:

Unstable angina (UA) and acute non-ST elevation myocardial infarction (NSTEMI) differ primarily in whether the ischemia is severe enough to cause sufficient myocardial damage to release detectable quantities of a marker of myocardial injury (troponins):

●UA is considered to be present in patients with ischemic symptoms suggestive of an ACS and no elevation in troponins, with or without electrocardiogram changes indicative of ischemia (eg, ST segment depression or transient elevation or new T wave inversion).

●NSTEMI is considered to be present in patients having the same manifestations as those in UA, but in whom an elevation in troponins is present.

MINS: Myocardial injury after non-cardiac surgery (up to 30 days post-op):
1. Elevated postop troponin
2. Resulting from myocardial ischaemia (i.e. no evidence of a non-ischaemic aetiology), not requiring an ischaemic feature (i.e. no chest pain, no ECG change)

VISION studies (Vascular Events in Noncardiac Surgery Patients Cohort Evaluation) demonstrated that severity of MINS strongly associated with 30-day mortality after NCS.

hs-cTnT
<20ng/L ~ 0.5% 30 day mortality
20-64ng/L ~3% 30 day mortality
65-999 ng/L ~9% 30 day mortality
>1000ng/L ~30% 30 day mortality

Whilst VISION trial identified MINS in at risk patients, the question now becomes what interventions are available to prevent this complication?

How well did you know this?
1
Not at all
2
3
4
5
Perfectly
58
Q

21.1 The 12 lead ECG shown (ECG with ST depression V1-V5, perhaps 1mm ste in lead 3) is most consistent with acute total occlusion of the

A. Posterior descending
B. RCA
C. LAD
D. OM

Note this is not the exact ECG with lead I changes too
A

C. LAD

How well did you know this?
1
Not at all
2
3
4
5
Perfectly
59
Q

20.2 A 46-year old man collapses unexpectedly and fractures his femur. He is booked for acute theatre. A pre-operative electrocardiogram is performed. Of the following, the most appropriate peri-operative medical management is (ECG shown)
ECG = WPW

a) Flecainide
b) Aspirin
c) Digoxin
d) Magnesium
e) Verapamil

A

a) Flecanide

WPW ECG = short PR, wide QRS, delta wave at start of QRS
If WPW, need to prolong refractor period of accessory pathway with agents such as procainamide/flecainide/amiodarone/sotalol.
Avoid verapamil (increases ventricular rate).
Avoid beta blockers (don’t affect accessory pathway).
https://litfl.com/wolff-parkinson-white-syndrome-ccc/

How well did you know this?
1
Not at all
2
3
4
5
Perfectly
60
Q

21.2 An electrocardiogram (ECG) abnormality which is NOT usually associated with severe anorexia nervosa is

a) Sinus tachycardia
b) Wandering atrial pacemakers
c) ST depression
d) T wave inversion
e) Prolonged QT

A

a) Sinus tachycardia

BJA: Anorexia nervosa: perioperative implications
https://academic.oup.com/bjaed/article/9/2/61/299563

Cardiovascular

Typically anorexic patients are hypotensive and bradycardic. These physiological markers may be used as indications for hospitalization.

Bradycardia reflects the decrease in basal metabolic rate that arises as an adaptive response to starvation. Although patients are usually in sinus rhythm, electrocardiographic abnormalities are common and may be found in >80% of strict dieters. These include: atrioventricular block, ST depression, T wave inversion, and QT prolongation.

QT prolongation may be caused by hypocalcaemia, hypomagnesaemia, drugs, or directly by starvation itself. Electrolyte disturbances have a significant causal role in ECG abnormalities.

Other factors, for example, atypical antipsychotics, may also contribute.

Associated arrhythmias include: sinus arrest, wandering atrial pacemakers, nodal escape beats, supraventricular tachycardia, and ventricular tachycardia.

The reported incidence of arrhythmias under anaesthesia is 16–62%.

With respect to myocardial contractility, left ventricular function has been demonstrated to be impaired in a proportion of patients. Echocardiographic studies have also demonstrated a higher incidence of mitral valve prolapse in anorexic patients. The reasons for this are not entirely clear. It is postulated that the loss of left ventricular volume and mass leads to abnormal mitral valve motion.

In addition to starvation-induced myocardial impairment, the myocardium may be specifically damaged by pharmacological agents.

For example, emetogenic ipecac syrup is directly cardiomyotoxic and produces inflammatory changes and myocardial fibre degeneration when used long term.

Rarely, antipsychotic drugs, for example, olanzapine, may cause cardiomyopathy.

Myocardial impairment can be caused by hypophosphataemia which also reduces the threshold for arrhythmias.

Compromised myocardial function requires judicious use of fluids perioperatively as there is an increased risk of congestive cardiac failure. Echocardiography along with invasive perioperative monitoring (central venous catheter) should be considered to prevent fluid overload.

How well did you know this?
1
Not at all
2
3
4
5
Perfectly
61
Q

21.2 A 69-year-old woman has a recent onset of dyspnoea and undergoes a right heart catheterisation, with results displayed below. Her pulmonary capillary wedge pressure is 10 mmHg. The most likely diagnosis is

a) Mitral stenosis
b) Mitral regurgitation
c) Aortic stenosis
d) Pulmonary embolism
e) COPD

A

d) Pulmonary embolism
- fits with history of acute dyspnoea
- PCWP normal, therefore precapillary PH
- thus left heart disease unlikely to be the cause of elevated RVSP (clinical group 2)
- COPD possible if cor pulmonale, but this is an unlikely cause of acute dyspnoea given history

Normal PCWP excludes left heart disease as cause of pulmonary HTN (so not MR, MS or AS). The causes of pre-capillary pulm HTN are pulmonary arterial hypertension, pulmHTN secondary to lung disease, chronic thromboembolic pulmonaryHTN, pulmHTN with unclear/multifactorial mechanisms.
Normal pulmonary capillary wedge pressure = 8- 12mmHg
Normal PASP: 15-25mmHg
Normal PADP: 8-15mmHg
Pulmonary HTN is mPAP ≥25mmHg at rest. mPAP = PADP + (PASP-PADP/3)
mPAP in this image is 43 mmHg Transpulmonary gradient = mPAP – PAWP
20B
20B

How well did you know this?
1
Not at all
2
3
4
5
Perfectly
62
Q

20.1 The radial artery pressure trace shown below is from a patient who has an intra-aortic balloon pump in situ. The device has been switched to 1:2 augmentation to assess the timing. The trace shows an augmented beat followed by an un-augmented beat. With respect to the augmentation, the trace shows

a. Correct timing
b. Early inflation
c. Late inflation
d. Early deflation
e. Late deflation

A

Bonus question

b. Early inflation

Waveform features:
> Diastolic augmentation (peak B) encroaches on the peak corresponding to unassisted systole (peak A) – the two peaks have merged and are barely distinguishable.
> There is no ‘sharp V’ or dicrotic notch between peaks A and B.

Early IAB inflation may result in:
> Premature closure of the aortic valve and possible aortic regurgitation, thus impairing left ventricular emptying. There may be an increase in LVEDV, LVEDP and PCWP.
> Increased left ventricular wall stress (afterload) and increased myocardial oxygen consumption will occur.

how to correct:
Delay the onset of IAB inflation, so that it inflates at the dicrotic notch resulting in a ‘sharp V’ (see the normal pressure waveform).

How well did you know this?
1
Not at all
2
3
4
5
Perfectly
63
Q

During a routine preoperative examination of a patient’s heart, you note exaggerated splitting of the second heart sound with inspiration. This is characteristically heard in

A. Aortic Reguritation
B. HOCM
C. Left bundle branch block
D. Mitral Stenosis
E. Pulmonary Stenosis

A

E. Pulmonary Stenosis

DERANGED PHYSIOLOGY:

Splitting of the first heart sound
Right bundle branch block can produce a split first heart sound - because the contraction of the right ventricle is delayed- the conduction occurs via the left ventricle rather than the bundle of His- and thefore the closure of the tricuspid valve occurs after a substantial delay.
Atrial septal defect can result in a fixed split of the first heart sound

Splitting of the second heart sound

It is normal for this sound to be split. The high pressure in the systemic circulation slams the aortic valve shut rather abruptly, almost angrily. In contrast, low pressure of the pulmonary circulation tends to close the pulmonary valve gently, and therefore the pulmonary component of the second heart sound (P2) is usually delayed by about 20-30 milliseconds.

It is also normal for increased right ventricular filling to cause a widening of the split. The more blood in the RV, the longer it takes to eject, and therefore the greater the delay until pulmonary valve closure.

n the spontaneously breathing patient, the delay is greatest during inspiration. Naturally, in the patient ventilated with positive pressure the delay is greatest during expiration (positive pressure being a barrier to diastolic filling).

Increased normal splitting of S2

Anything that delays the end of right ventricular systole can cause this sort of picture.

Right bundle branch block - the delay in conduction via the left ventricle causes a delay in right ventricular contraction, and therefore a delay in pulmonary valve closure. The S1 will also be split.
Ventricular septal defect - because the right ventricle receives a large volume load directly from the left ventricle, and therefore takes longer to complete its systolic contraction.

Pulmonary valve stenosis - because the right ventricle takes longer to empty though a narrowed valve

Mitral regurgitation- not because right ventricular contraction is delayed, but because left ventricular contraction is shortened (as the LV empties in both the aortic and the atrial directuion, systole is over very quickly).

Fixed splitting of S2

Atrial septal defect - the atria, joined by a gaping hole in their seput, act as one atrium. The result is a reasonably equal distribution in volume betweent the right and left atrium. This way, both sides of the circulation share the same diastolic filling pressure. Dragging more volume into the right atrium with respiratory activity will not cause an inequality of ventricular filling (between the right and left ventricles) because the venous return will be “shared”.

Reversed splitting of S2

In this situation, P2 occurs before A2, and splitting widens during expiration (or inspiration in the mechanically ventilated patient). This only happens if the conduction to the left ventricle is delayed, or if the left ventricle is massively volume overload (and the right ventricle is not).
Left bundle branch block - the left ventricle depolarises after the right ventricle, and A2 is delayed
Aortic stenosis - the left ventricle empties slowly though a narrow valve
Large patent ductus arteriosus - the left ventricle receives a backflow of blood from the aorta, which causes it to become volume-overloaded

How well did you know this?
1
Not at all
2
3
4
5
Perfectly
64
Q

22.2 A 45-year-old man is ventilated in the intensive care unit and is in a critical state. His pulmonary artery wedge pressure is 26 mmHg, cardiac index is 1.7 L/minute/m2 and his PaO2/FiO2 ratio is 200 mmHg. A decision is made to place him on extracorporeal membrane oxygenation. The most appropriate mode is

a) VV ECMO
b) VA ECMO
c) Atrio-aorto ECMO
d) Ventriculo-atrial ECMO

A

b) VA ECMO

PaO2/FiO2 ratio
Mild: 200-300 = mortality 27%
Moderate = 100-200 mortality 32%
Severe < 100 = Mortality 45%

Cardiac Index
Normal: 2.5-4.2l/min

PAWP:
Normal 4-12mmHg

CI is low, PaO2/FiO2 ratio is mild, PAWP is high

How well did you know this?
1
Not at all
2
3
4
5
Perfectly
65
Q

23.1 A third heart sound at the apex may be heard in

a) pulmonary stenosis
b) pulmonary hypertension
c) pericarditis
d) pregnancy

A

d. pregnancy

How well did you know this?
1
Not at all
2
3
4
5
Perfectly
66
Q

21.2 A patient presents for a trans-urethral resection of the prostate (TURP). He had a single drug-eluting coronary stent for angina pectoris inserted six months ago and is taking clopidogrel and aspirin. The most appropriate preoperative management of his medications is to

a) Cease aspirin, continue clopidogrel
b) Cease aspirin for 10 days, cease clopidogrel for 5 days
c) Cease clopidogrel for 5 days, continue aspirin
d) Cease clopidogrel for 10 days, continue aspirin
e) Continue both aspirin and clopidogrel

A

c) Cease clopidogrel for 5 days, continue aspirin
- prostatic surgery, the risk of major bleeding may be greater than the risk of stent thrombosis
- For clopidogrel, we stop five days before surgery
- Clopidogrel, if stopped, should be restarted with a loading dose of 300 mg as soon as possible after surgery, perhaps later in the day if postoperative bleeding has stopped. Some experts recommend a higher loading dose of 600 mg to decrease time to effectiveness in the higher-risk postoperative setting
- suggest that surgery be performed in centers with 24-hour interventional cardiology coverage

UP TO DATE: Noncardiac surgery after PCI

Nonemergency noncardiac surgery — For patients who have undergone previous stenting with either BMS or DES and who will need cessation of one or both antiplatelet agents, we prefer to defer planned nonemergency, nonurgent noncardiac surgery until at least six months after stent implantation. The risks of noncardiac surgery before six months are increased after both BMS and DES.
For patients whose surgery requires cessation of one or both antiplatelet agents and cannot wait six months, and where the risks of delaying surgery outweigh the benefits, our recommended minimal duration of DAPT is four to six weeks, depending on the urgency of surgery and risk of thrombotic complication. This is based in part on evidence suggesting that the increased risk of MI and cardiac death is highest within the first month after stent placement and no clear difference in risk between BMS and DES. Although we prefer to wait at least six weeks when possible, in patients for whom earlier surgery is in their best interest after weighing risks and benefits, we sometimes refer patients as early as four weeks after stent placement.

The proinflammatory and prothrombotic risks of surgery may increase the baseline risk of stent thrombosis even in the presence of DAPT and regardless of stent type during this early period after stenting. We believe this risk to be higher prior to the minimum duration of DAPT recommended above, but the final decision to continue or discontinue antiplatelet therapy in the perioperative period should be made only after an informed discussion among the surgeon, managing cardiologist (and other health care providers), and patient has taken place. In many cases, DAPT can be continued in the perioperative period, although for some surgeries, such as neurosurgery, posterior eye surgery, or prostatic surgery, the risk of major bleeding may be greater than the risk of stent thrombosis.

In these patients who undergo noncardiac surgery before the recommended minimum duration of DAPT, a platelet P2Y12 receptor blocker should be discontinued for as brief a period as possible. Aspirin should be continued through the perioperative period, since the risk of stent thrombosis is further increased with the cessation of both aspirin and clopidogrel and surgery can usually be safely performed on aspirin. The rationale to continue aspirin comes in part from the POISE-2 trial (PCI subgroup analysis), which is discussed separately. However, as many neurosurgical patients, for whom bleeding might be life threatening or lead to severe adverse outcomes, were not enrolled in POISE-2, the optimal strategy is not known.

●Minor surgical and dental procedures usually do not require cessation of antiplatelet therapy.
●With regard to stopping P2Y12 inhibitor prior to noncardiac surgery, we generally follow recommendations found in the manufacturer’s package insert for each drug.
- For clopidogrel, we stop five days before surgery; that is, the last dose is taken on the sixth day before surgery.
- For prasugrel, we stop seven days before surgery.
- For ticagrelor, we stop three to five days before surgery.
- Some experts are willing to recommend shorter discontinuation periods for procedures less likely to be associated with major bleeding.
●Clopidogrel, if stopped, should be restarted with a loading dose of 300 mg as soon as possible after surgery, perhaps later in the day if postoperative bleeding has stopped. Some experts recommend a higher loading dose of 600 mg to decrease time to effectiveness in the higher-risk postoperative setting.
●We suggest that surgery be performed in centers with 24-hour interventional cardiology coverage

How well did you know this?
1
Not at all
2
3
4
5
Perfectly
67
Q

23.1 A patient presents for a transurethral resection of the prostate (TURP). He had a single drug-eluting coronary stent for angina pectoris inserted six months ago and is taking clopidogrel and aspirin. The most appropriate preoperative management of his medications is to

a) Cease aspirin, continue clopidogrel
b) Cease aspirin for 10 days, cease clopidogrel for 5 days
c) Cease clopidogrel for 5 days, continue aspirin
d) Cease clopidogrel for 10 days, continue aspirin
e) Continue both aspirin and clopidogrel

A

C) Cease clopidogrel for 5 days, continue aspirin

WFSA update document
https://resources.wfsahq.org/wp-content/uploads/uia29-Perioperative-management-of-patients-with-coronary-stents-for-non-cardiac-surgery.pdf

Dual antiplatelet therapy should be continued in all patients with coronary stents presenting for surgery.

However, if there is a high risk of surgical bleeding then clopidogrel should be stopped 5-7 days before surgery and monotherapy with aspirin should be continued.

Clopidogrel should be restarted as soon as possible post surgery. Cessation of aspirin therapy may be considered during intracranial surgery and transuretheral resection of prostrate as these procedures are associated with an increased risk of bleeding, but only after contemplating the risk-benefit ratio.

2014 AHA/ACC guidelines on perioperative medicine don’t give a firm answer except: > 180 days since insertion = proceed (Level II b evidence)

How well did you know this?
1
Not at all
2
3
4
5
Perfectly
68
Q

22.1 A 26-year-old patient presents with exertional syncope. The most likely diagnosis is

a. HOCM
b. Long QT syndrome
c. CCF
d. IHD

A

HOCM: pathopneumonic

A person who has syncope during exertion is more likely to have an obstruction to blood flow (aortic stenosis or hypertrophic cardiomyopathy) or ventricular tachycardia as a cause. On the other hand, syncope after completion of exercise is more likely of reflex origin, such as the common faint.

https://www.uptodate.com/contents/syncope-fainting-beyond-the-basics#:~:text=A%20person%20who%20has%20syncope,such%20as%20the%20common%20faint

How well did you know this?
1
Not at all
2
3
4
5
Perfectly
69
Q

23.1 You are called to assist in the resuscitation of a 75-year-old patient in the emergency
department who is in extremis with severe hypotension and hypoxaemia. The image
shown is of a focused transthoracic echocardiogram, parasternal short axis view.
The most likely diagnosis is

a. PE
b. Tamponade

A

a) PE

D-shaped left ventricle

How well did you know this?
1
Not at all
2
3
4
5
Perfectly
70
Q

22.2 You are inserting a pulmonary artery catheter in an intubated patient prior to cardiac surgery and a significant amount of blood appears in the endotracheal tube. The most appropriate specific initial management is to

a. Remove PAC and insert DLT
b. Wedge PAC and insert BB
c. Wedge PAC and insert DLT
d. Withdraw PAC 2cm and insert DLT

A

d. Withdraw PAC 2cm and insert DLT

Pulmonary rupture

Miller:
- Position pt with bleeding lung dependent
- Perform endotracheal intubation, oxygenation, airway toilet
- Isolate lung by endobronchial DLT or SLT or bronchial blocker
- Withdraw PAC several centimetres, leaving it in the main PA. Do not inflate the balloon (except with fluoroscopic guidance)
- Position pt with isolated bleeding lung nondependent. Administer PEEP to the bleeding lung if possible
- Transport the patient to medical imaging for diagnosis and embolisation if feasible

How well did you know this?
1
Not at all
2
3
4
5
Perfectly
71
Q

The maximum warm ischaemic time acceptable for procuring the kidney following donation after cardiac death is

A. 30 mins
B. 60 mins
C. 90 mins
D. 120 mins
E. 180 mins

A

Warm ischaemia time:
- Time from treatment withdrawal to the start of cold perfusion of the donated organs
- Significance is the impact on graft function
- Most important phase of WIT begins when the systolic BP is < 60mmHg
- This includes the waiting period from the absence of circulation to the declaration of death and the time before initiating the flow of cold perfusate through the cannula

Maximum WARM Ischaemia time
- Heart 30 mins
- Liver 30 mins
- Pancreas 30 mins
- Kidney 60 mins
- Lungs 90 mins

Maximum COLD Ischaemia time:
- Heart = 4 hrs
- Lungs = 6-8hrs
- Liver/Pancreas = 12hrs (DBD)/6 hrs (DCD)
- Kidneys = 18hrs (DBD)/ 12 hrs (DCD)

72
Q

21.2 Cardiovascular effects of hyperthyroidism include

a) Increased DBP
b) Narrow pulse pressure
c) Reduced diastolic relaxation
d) Decreased CO
e) Decreased SVR

A

e) Decreased SVR
- increased CO, increased SBP and decreased DBP with widened PP

UP TO DATE: Cardiovascular effects of hyperthyroidism:

  • Thyroid hormone has important effects on cardiac muscle, the peripheral circulation, and the sympathetic nervous system that alter cardiovascular hemodynamics in a predictable way in patients with hyperthyroidism.
  • The main changes are :
    ●Increases in heart rate, cardiac contractility, systolic and mean pulmonary artery pressure, cardiac output, diastolic relaxation, and myocardial oxygen consumption
    ●Reductions in systemic vascular resistance and diastolic pressure
73
Q

23.1 A patient is suffering an acute myocardial infarction. Australian and New Zealand
guidelines recommend the threshold for the use of supplemental oxygen is when the
SpO2 falls below

a. 88%
b. 90%
c. 93%
d. 97%
e. 100%

A

c) 93%

ANZCOR suggests against the routine administration of oxygen in persons with chest pain.13 [2015 COSTR, weak recommendation, very-low certainty evidence]
For persons with heart attack, routine use of oxygen is not recommended if the oxygen saturation is >93% [National Heart Foundation of Australia & Cardiac Society of Australia and New Zealand: practice advice].9

74
Q

20.1 The radial artery pressure trace shown below is from a patient who has an intra-aortic balloon pump in situ. The device has been switched to 1:2 augmentation to assess the timing. The trace shows an augmented beat followed by an un-augmented beat. With respect to the augmentation, the trace shows

a. Correct timing
b. Early inflation
c. Late inflation
d. Early deflation
e. Late deflation

A

d. Early deflation

https://litfl.com/intra-aortic-balloon-pump-trouble-shooting/

Waveform features:
> There is a sharp drop in pressure immediately following the peak of diastolic augmentation (peak B).
> Diastolic augmentation may be suboptimal but it is difficult to confirm in the absence of a pressure scale or comparison to an unassisted waveform.
> Assisted aortic end-diastolic BP may be sub-optimally increased (trough C), but it is difficult to say based on the information given. With early deflation a widened U-shaped trough is typically seen.
> Early deflation can lead to an assisted aortic end-diastolic BP that equals or exceeds the unassisted aortic end-diastolic BP (trough F), although this is not the case in this scenario.
> Assisted SBP (peak D) is the same or higher than the unassisted SBP (peak A) – it should be slightly less.

Early IAB deflation may result in:
> inadequate coronary perfusion, with the potential for retrograde coronary blood flow. This may result in angina due to decreased myocardial oxygen supply.
> suboptimal afterload reduction and increased myocardial oxygen demand

How to correct:
Prolong the IAB inflation time, so that it deflates at the end of diastole, just before the onset of isovolumetric systolic contraction.

75
Q

20.2 You are called to assist with a patient in the intensive care unit who has had cardiac surgery three days ago and is now in cardiac arrest. External cardiac massage should aim for a systolic blood pressure of

a. 40
b. 60
c. 80
d. 100
e. 120

A

b. 60

BJA Article - ​Management of cardiac arrest following cardiac surgery - BJA Education

In the CICU, the effectiveness of ECC is confirmed by monitoring the arterial pressure trace with a target compression rate and depth to achieve a systolic impulse of > 60 mm Hg to maintain a mean perfusion pressure, preventing ventricular distension, LV wall stress, and ischaemia.

76
Q

21.2 The most common complication of extracorporeal membrane oxygenation (ECMO) in adults is

a) Bleeding
b) Vascular damage
c) Embolism
d) Inadvertently decannulate

A

a) Bleeding

ECMO complications:
- patient complications: bleeding & coagulopathy most common
- mechanical complications: access insufficiency common

Blue book 2017

77
Q

22.2 In a previously normal patient with cardiac failure secondary to acute pulmonary embolism, the best choice of vasoactive agent for initial treatment is

a. Dobutamine
b. Milrinone
c. Isoprenaline
d. Noradrenaline

A

d. Noradrenaline

Supportive Management of Massive PE

Coexisting left ventricular systolic dysfunction and diastolic dysfunction complicate the management of heart failure patients with massive PE. Although a common strategy in response to systemic arterial hypotension is to prescribe a fluid bolus, volume loading may worsen biventricular failure, pulmonary edema, and hypoxemia. An initial trial of volume expansion, limited to 250 to 500 mL, may be attempted in those heart failure patients without evidence of increased right-sided filling pressures or pulmonary edema.6

Although non–heart failure patients generally respond well to pure vasopressors for hemodynamic support in massive PE, many heart failure patients will not tolerate the isolated increase in systemic vascular resistance. PE patients with heart failure may require an agent with mixed vasopressor and inotropic properties such as norepinephrine, epinephrine, or dopamine. Whereas LV function often becomes hyperdynamic to compensate for RV failure, the presence of underlying LV systolic dysfunction in heart failure patients may limit the patient’s ability to maintain normal systemic cardiac output and may necessitate the addition of inotropes.

https://www.ahajournals.org/doi/10.1161/CIRCULATIONAHA.108.803965

78
Q

20.1 Infrarenal aortic cross clamp will cause a(an):

a) Increased by 40% renal blood flow
b) Increased by 20% renal blood flow
c) Unchanged renal blood flow
d) Decreased 20% renal blood flow
e) Decreased 40%renal blood flow

A

e) Decreased 40%

Infra-renal aortic cross-clamping leads to a reduction in renal blood flow by up to 40%, as a result of an increase in renal vascular resistance of up to 75%.

The mechanism underlying this increased resistance is uncertain but may, in part, be a result of the associated decrease in cardiac output during aortic cross-clamping, as well as because of humoral mechanisms, which lead to increased release of renin. After declamping, there is a maldistribution of renal blood flow away from the cortex for at least 60 min.

79
Q

22.1 Of the following, the drug most likely to cause pulmonary arterial vasodilation with systemic arterial vasoconstriction when used in low doses is

a) Adrenaline
b) Noradrenaline
c) Vasopressin
d) Dopamine
e) Dobutamine

A

c) Vasopressin

https://emcrit.org/ibcc/pressors/

  • From UP TO DATE:
    > At low doses of 1 to 3 mcg/kg per min, dopamine acts primarily on dopamine-1 receptors to dilate the renal and mesenteric artery beds
    > At 3 to 10 mcg/kg per min (and perhaps also at lower doses), dopamine also stimulates beta-1 adrenergic receptors and increases cardiac output, predominantly by increasing stroke volume with variable effects on heart rate.
    > At medium-to-high doses, dopamine also stimulates alpha-adrenergic receptors, although a small study suggested that renal arterial vasodilation and improvement in cardiac output may persist as the dopamine dose is titrated up to 10 mcg/kg per min
    *clinically, the haemodynamic effects of dopamine demonstrate individual variability

Dobutamine (inodilator):
- selective β1-agonist that increases cardiac contractility and reduces pulmonary vascular and systemic vascular resistances

Vasopressin:
- vasopressin may have pulmonary vasodilatory effects in addition to a systemic vasoconstrictive effect

Milrinone (inodilator):
- the phosphodiesterase-3 inhibitors, milrinone and enxoimone, have positive inotropic effects combined with the capacity to reduce RV afterload (‘inodilators’) without significant chronotropic effect, but they can be associated with significant systemic hypotension

80
Q

20.1 Patient with Fontan circulation and peritonism having induction for laparotomy. Drops sats on induction. Best move?

a. Decrease volatile
b. Reverse Trendelenberg
c. Decrease FiO2
d. Increase PEEP
e. Increase tidal volume

A 22-year-old man with a Fontan circulation is on your emergency list for an appendicectomy. He has had abdominal pain and vomiting for 3 days, and has a peritonitic abdomen. His preoperative arterial oxygen saturation is 95%. Shortly after induction he becomes hypotensive BP 80/45, and saturations fall to 75%. His condition is most likely to be improved by:

A. Increasing the inspiratory time.
B. Decreasing the ventilator tidal volumes.
C. Adding positive end-expiratory pressure (PEEP).
D. Positioning reverse trendelenberg.

A

A

Couldn’t find a clear source but we know;

A - will decrease venoplegia and improve venous return
B - Would not help, decrease VR
C - Don’t drop FiO2 when desatting…
D - increases PVR (unless below FRC) and reduces pulmonary flow
E - Same as above, increased PVR and reduces flow through pulmonary circuit

B. Decreasing the ventilator tidal volumes.

Patients who have undergone the Fontan procedure depend on blood flow through the pulmonary circulation without the assistance of the right ventricle. The difference between central venous pressure and systemic ventricular end-diastolic pressure (termed the “transpulmonary gradient”) is the primary force promoting pulmonary blood flow and, more importantly, cardiac output.

Circulation in the Fontan patient is promoted by low pulmonary vascular resistance. Positive-pressure ventilation with increased tidal volumes, as described above, can result in excessive intrathoracic pressures, leading to decreased venous return to the heart and increased pulmonary vascular resistance.
In periods of low oxygen saturation, 100% inspiratory oxygen is appropriate.
The addition of PEEP will increase intrathoracic pressure, reducing venous return.
Trendelenberg positioning would increase CVP and therefore bloodflow through pulmonary circulation.

81
Q

22.1 When using cardioversion to revert a patient in atrial fibrillation to sinus rhythm, the direct current shock is synchronised with the ECG to coincide with the

a. Start of R wave
b. Start of Q wave
c. Middle of T wave
d. peak of R wave

A

R-wave peak

82
Q

21.1 A 25-year-old man suffers a 30% total body surface area burn. A cardiovascular physiological change expected within the first 24 hours is

a. Decreased PVR
b. Increased SVR
c. Decreased SVR
d. Reduced PA pressure
e. Increased hepatic blood flow

A

increased SVR

EMSB handbook
CO is reduced after Burn injury 2ry to:
- myocardial depressant mediators
- decreased blood volume
- reduced venous return
- increased pulmonary and systemic vascular resistance due to increased levels of catecholamines

In the first 24hrs reduced cardiac output persists even after restoration of blood volume

Between 24-48hrs post burn a hyperdynamic state develops with reduced peripheral resistance, increased oxygen consumption and increased cardiac output

83
Q

23.1 Pulse pressure variation is defined as

a. 100x SBP max - SBP min / SBP min
b. 100 x PPmax - PPmax / PPmin
c. 100x SBP max - SBP min/ SBP mean
d. 100 x PPmax - PPmin / PPmean

A

d) 100 x PPmax - PPmin / PPmean

84
Q

20.2 This ECG rhythm strip represents

A
85
Q

22.2 AA 15-year-old patient with a known prolonged QT interval has a ventricular tachyarrhythmia while being monitored postoperatively in the postanaesthesia care unit. The patient is alert, orientated and without chest pain but feels unwell. The best initial management is

A. Magnesium
B. Synchronised shock
C. Amiodarone
D. Adenosine
E. Metoprolol

A

A. Magnesium TdP

UTD - BB specifically propanolol or nadolol
if Hx of syncope / seizures or resus SCA

https://www.uptodate.com/contents/congenital-long-qt-syndrome-treatment

86
Q

22.1 Abnormal Q waves are NOT a feature of the electrocardiogram in

A. Digitalis toxicity
B. LBBB
C. Recent transmural MI
D. Wolff-Parkinson-White
E. Previous MI

A

A. Digitalis toxicity

Miller’s
The ECG made easy
http://lifeinthefastlane.com/ecg-library/pmi/

Normal Q waves
- Due to depolarisation of the interventricular septum from left to right
- Seen in the left-sided leads (I, aVL, V5, V6)

Pathological Q waves
- > 1 mm depth
- > 1 mm (= 40 ms) across

Digoxin ECG changes
- Therapeutic: prolonged PR interval (AV nodal delay), shortened QTC intervals (rapid ventricular repolarisation), ST depression (↓ slope of phase 3), T wave inversion
- Toxic: atrial or ventricular arrhythmias (↑ automaticity), prolonged PR interval → heart block, SA node inhibition → sinus arrest
- Atrial tachycardia with block = most common arrhythmia attributed to digoxin toxicity
- VF = most frequent cause of death
- QRS = normal!

Q waves in MI
- Occur with transmural infarctions, and are less likely with subendocardial infarctions
- Develops days after the onset of AMI, and is usually permanent
- Indicates the part of the heart that has been damaged

LBBB ECG changes
- Wide QRS
- Wide QS complex in lead V1
- Wide R wave in lead V6 with slight notching at the peak and TWI
- The axis is highly variable: can be normal or deviated to the left or right

Wolff-Parkinson-White syndrome
- Due to the presence of an accessory bundle between the atrium and ventricle, which has no AV node to delay conduction
- Short PR interval
- Early slurred upstroke of the QRS complex due to delta wave

87
Q

23.1 This Doppler trace obtained by transoesophageal echocardiography of the descending aorta suggests

a. AS
b. AR

A

b. AR

https://litfl.com/oesophageal-doppler/

88
Q

23.1 A multitrauma patient is being managed with a resuscitative endovascular balloon occlusion device of the aorta (REBOA) as part of damage control resuscitation. The
balloon has been inserted for intractable pelvic bleeding. The most appropriate location for the device placement is between the

A. Between artery of adamkiewicz to coeliac artery
B. Between coeliac artery to renal artery
C. Between lowest renal artery to bifurcation of aorta
D. Between coeliac and bifurcation

A

C. Between lowest renal artery to bifurcation of aorta

https://litfl.com/reboa-in-resuscitation/

Anatomy:
The aorta is divided into three separate zones for the purposes of REBOA (aortic length varies between individuals)

Zone I of the aorta extends from the origin of the left subclavian artery to the coeliac artery (approx 20cm long in a young adult male)
Zone II extends from the coeliac artery to the most caudal renal artery (approx 3cm long)
Zone III extends distally from the most caudal renal artery to the aortic bifurcation (approx 10cm long)

REBOA location based on injury:

> suspected or diagnosed intra-abdominal haemorrhage due to blunt trauma or penetrating torso injuries (Zone I REBOA), or

> blunt trauma patients with suspected pelvic fracture and isolated pelvic haemorrhage (Zone III REBOA), or

> patients with penetrating injury to the pelvic or groin area with uncontrolled haemorrhage from a junctional vascular injury (iliac or common femoral vessels) (Zone III REBOA)

Simplistic rendering of aorta. Zone 1 (from left subclavian artery to the upper border of the celiac trunk), Zone 2 (the upper border of the celiac trunk to the lower border of the distal take-off of the renal arteries), and Zone 3 (from the lower border of the lower renal artery to the aortic bifurcation). Zone 1 is occluded in the case of cardiac arrest or life-threatening intra-abdominal hemorrhage; Zone 2 has no current indication; and Zone 3 is occluded in the case of life-threatening pelvic or lower limb haemorrhage7. REBOA Resuscitative Endovascular Balloon Occlusion of the Aorta.

89
Q

20.1 The radial artery pressure trace shown below is from a patient who has an intra-aortic balloon pump in situ. The device has been switched to 1:2 augmentation to assess the timing. The trace shows an augmented beat followed by an un-augmented beat. With respect to the augmentation, the trace shows

a. Correct timing
b. Early inflation
c. Late inflation
d. Early deflation
e. Late deflation

A

Bonus question

e. Late deflation

https://litfl.com/intra-aortic-balloon-pump-trouble-shooting/

Waveform features:
> The peak corresponding to diastolic augmentation (peak C) is widened.
> Assisted aortic end-diastolic BP (trough E) is the same as, not lower than, the unassisted aortic end-diastolic BP (trough G).
> The upstroke of assisted systolic BP (peak F) has a gentle gradient resulting in a prolonged rise.

Late deflation of the IAB has these effects:
> There is no afterload reduction. The inflated balloon may actually impede left ventricular ejection and increase the afterload.
> Myocardial oxygen consumption will increase because the left ventricle experiences a longer period of isovolumetric contraction (when most myocardial oxygen consumption occurs) and has to contract against greater resistance (afterload).

How to correct:
> Shorten the IAB inflation time, so that the IAB deflates at the end of diastole – just before isovolumetric contraction of the left ventricle.

90
Q

23.1 A patient with idiopathic pulmonary hypertension has had a right heart catheter with
the following results The transpulmonary gradient is

(table of numbers from RHC given, including mPAP 40 and PCWP 13)

A

? no recalled ?

MPAP – PCWP = Transpulmonary gradient

27mmHg

TPG = mPAP – PCWP

91
Q

21.2 The maximum warm ischaemia time acceptable for procuring the kidney following donation
after circulatory death (DCD) is

a) 30 minutes
b) 60 minutes
c) 90 minutes
d) 120 minutes

A

Warm ischaemia time:
- Time from treatment withdrawal to the start of cold perfusion of the donated organs
- Significance is the impact on graft function
- Most important phase of WIT begins when the systolic BP is < 60mmHg
- This includes the waiting period from the absence of circulation to the declaration of death and the time before initiating the flow of cold perfusate through the cannula

Maximum WARM Ischaemia time
- Heart 30 mins
- Liver 30 mins
- Pancreas 30 mins
- Kidney 60 mins
- Lungs 90 mins

Maximum COLD Ischaemia time:
- Heart = 4 hrs
- Lungs = 6-8hrs
- Liver/Pancreas = 12hrs (DBD)/6 hrs (DCD)
- Kidneys = 18hrs (DBD)/ 12 hrs (DCD)

92
Q

21.1 Cardiovascular effects of hyperthyroidism include

a. Decreased SVR
b. Increased SVR
c. Decreased diastolic relaxation
d. Decreased PVR
e. Increased diastolic blood pressure

A

decreased SVR

Hyperthyroidism causes a hyperdynamic circulation, characterized by increased cardiac contractility and heart rate, increased preload, and decreased systemic vascular resistance (SVR), resulting in significantly increased cardiac output

https://www.sciencedirect.com/science/article/pii/S0735109718333795#:~:text=Hyperthyroidism%20causes%20a%20hyperdynamic%20circulation,in%20significantly%20increased%20cardiac%20output.

93
Q

21.2 A 59-year-old lady presents for elective coronary artery graft surgery. She has a pulmonary artery catheter inserted with the waveforms displayed below. Her cardiac output is 4.5 L/min.

Her mean pulmonary artery pressure is 33 mmHg. The most likely explanation for the waveforms seen is that she has

A

MR?? See UpToDate PAC interpreting

94
Q

21.1 A 30-year-old professional athlete who underwent a knee arthroscopy under general anaesthesia becomes tachycardic in the recovery room. His non-invasive systolic blood pressure is 90 mmHg. A 12-lead ECG is obtained. The most appropriate therapy is

a. Adenosine 6mg (or 60mg remembered by other cohort)
b. valsalva
c. 50J
d. 200J

A

b. valsalva

Fluid and magnesium - fixes all.

But could also be conscious VT or something stupid….

95
Q

20.1 Differential hypoxia is seen during what?

a) Ecco2 device
b) VV ecmo
c) VA ecmo
d) Haemodialysis
e) Peritoneal Dialysis

A

VA ecmo

VA - bleeding (large bore arterial puncture)

96
Q

20.2 The right ventricular systolic pressure (RVSP) can be used to estimate pulmonary arterial pressure. The method used for determining RVSP by echocardiography involves applying the

a) Poiseuille equation, tricuspid regurg peak + RAP
b) Poiseuille equation using mitral peak + LAP
c) Poiseuille equation using mitral peak + CVP
d) Bernoulli equation using tricuspid regurg peak + LAP
e) Bernoulli equation using tricuspid regurg peak + RAP

A

Answer: e) Bernoulli equation using tricuspid peak + RAP
i.e. RVSP = 4v2 + RAP (~PAP in absence of RVOTO)

97
Q

21.2 You are examining the precordium of a patient in the preadmission clinic and hear a fourth heart sound at the apex. This finding is consistent with

a) AR
b) Athlete
c) Normal
d) Hypertension

A

) Hypertension

Talley & O’Connor CVS Exam:
S3: Physiological in pregnancy; sign of LV failure; AR & MR
S4: Never physiological, most often due to systemic hypertension

Atrial gallop - stiff LV
- hypertrophy or ischaemic ventricle

Source CV phys

98
Q

20.1 ECG with infarct what territory

a) PDA
b) Obtuse marginal
c) LAD
d) RCA
e) Left circumflex

A

RCA

Source: LITFL

RCA occlusion is suggested by:
ST elevation in lead III > lead II
Presence of reciprocal ST depression in lead I
Signs of right ventricular infarction: STE in V1 and V4R

99
Q

22.1 A 75-year-old man has a loud ejection systolic murmur detected on clinical examination before a joint replacement. A focused transthoracic echocardiogram (TTE) detects a calcified aortic valve with a peak aortic jet velocity of 3 m/s. The peak gradient across the aortic valve is

a) 36mmHg
b) 44mmHg
c) 48mmHg
d) 56mmHg
e) None of the above

A

= ΔP = 4v2 = 4 x 9 = 36

100
Q

22.2 The most common complication of extracorporeal membrane oxygenation (ECMO) in adults is

a. Bleeding
b. Thrombosis
c. infection
d. gas embolism

A

a. Bleeding

repeat

101
Q

22.1 A 70-year-old man undergoes a stress echocardiogram as part of his preoperative preparation before a total hip replacement. If he has clinically significant coronary artery disease, the earliest indicator during his test is most likely to be

a. Angina
b. ECG changes
c. Hypotension
d. RWMA

A

RWMA

Echocardiographic images recorded during ischemia show abnormalities of wall motion, thus allowing for the detection of significant coronary artery disease
https://www.sciencedirect.com/topics/nursing-and-health-professions/stress-echocardiography

102
Q

21.2 The apical four–chamber view of a transthoracic echocardiogram below shows

a. PFO
b. SAM
c. Dilated RA and RV
d. Dilated LA and LV
e. Tamponade

A

Dilated RA and RV

103
Q

20.1 The radial artery pressure trace shown below is from a patient who has an intra-aortic balloon pump in situ. The device has been switched to 1:2 augmentation to assess the timing. The trace shows an augmented beat followed by an un-augmented beat. With respect to the augmentation, the trace shows

a. Correct timing
b. Early inflation
c. Late inflation
d. Early deflation
e. Late deflation

A

Bonus question

a. Correct timing

https://derangedphysiology.com/main/required-reading/cardiothoracic-intensive-care/Chapter%20634/normal-iabp-waveform

104
Q

20.1 A 34-year-old woman with cystic fibrosis has had a recent transthoracic echocardiogram to evaluate pulmonary pressure and suitability for lung transplantation. Below is a continuous wave Doppler trace through the tricuspid valve.
Peak velocity= 3
Her central venous pressure is 5 mmHg. Her estimated right ventricular systolic pressure (RVSP) is

a) 39
b) 41
c) 45
d) 50
e) 61

A

a) 41mmHg

Answer: RVSP = 4v2 + CVP
4x3x3+5 = 41

105
Q

22.1 You review a patient before major bowel surgery. Using the American Heart Association/American College of Cardiology consensus guidelines, you assess him as being at intermediate risk of a perioperative adverse cardiac event. When explaining this to the patient, this best translates to a numerical risk in the range of

a) 1-5%
b) 5-10%
c) 10-15%
d) 15-20%

A

a) <1% low risk for 30 day adverse cardiovascular event. >5% high. 1-5% therefore moderate.

https://www.ahajournals.org/doi/10.1161/circ.105.10.1257

Based on surgery type

c) 10-15% (unlikely this)

Based on patient factors alone, adults can be categorized into low (<5%), borderline (5 to <7.5%), intermediate (≥7.5 to <20%), or high (≥20%) 10-year CVD risk. Source: ACC/AHA Guideline 2019

https://www.acc.org/latest-in-cardiology/ten-points-to-remember/2019/03/07/16/00/2019-acc-aha-guideline-on-primary-prevention-gl-prevention

https://www.jacc.org/doi/epdf/10.1016/j.jacc.2019.03.010

106
Q

20.1 A 55-year-old man with no past history of ischaemic heart disease is 3 days post total hip replacement surgery. He has an episode of chest pain that sounds ischaemic, began at rest and lasts thirty minutes before resolving fully. There are no ECG changes. 6 hours later there is a troponin rise above the 99th percentile upper reference limit. The diagnosis is

A. STEMI
B. NSTEMI
C. Myocardial injury for non cardiac surgery (MINS)
D. No diagnosis
E. Unstable angina

A

C. MINS (Anthony reckons)

B. NSTEMI

https://resources.wfsahq.org/atotw/perioperative-myocardial-ischaemia-in-non-cardiac-surgery/

MI is defined as myocardial cell death due to prolonged myocardial ischaemia. It is diagnosed by22 a rise of cardiac biomarker value above the 99th percentile limit with at least 1 of the following:
1. Symptoms of ischaemia,
2. New ST-segment T wave changes or new left bundle branch block,
3. New pathological Q waves,
4. Imaging evidence of new loss of viable myocardium or new regional wall motion abnormality
5. Identification of an intracoronary thrombus by angiography or autopsy OR: Cardiac death with symptoms suggestive of myocardial ischaemia.

It is important to note that the above diagnostic criteria have not been created for the perioperative setting. An elevated troponin after noncardiac surgery, even without other features of ischaemia independently increases the risk of 30-day mortality.21 With this in mind, a new perioperative diagnosis has been created—myocardial injury after noncardiac surgery (MINS). MINS is diagnosed by a postoperative peak troponin T of 0.03 ng/mL or greater due to myocardial ischaemia.21 Although a diagnosis of MINS has prognostic significance, the clinical utility remains uncertain.

MINS is defined as myocardial cell injury during the first 30 days after noncardiac surgery due to an ischemic etiology (ie, no evidence of a nonischemic etiology like sepsis, rapid atrial fibrillation, pulmonary embolism, cardioversion, chronically elevated troponin, etc) and is independently associated with mortality. MINS includes MI (both symptomatic and non-symptomatic) and patients with postoperative elevations in troponin but who do not have symptoms, electrocardiographic abnormalities, or other criteria that meet the universal definition described above, and have no evidence of a nonischemic etiology for their troponin elevation)

Circulation https://www.ahajournals.org/doi/10.1161/CIR.0000000000001024
MINS includes myocardial infarction and ischemic myocardial injury that do not fulfill the Universal Definition of Myocardial Infarction (myocardial injury with a rise or fall of cTn above the 99th percentile of the upper reference limit and at least 1 of the following: ischemic symptoms, new ischemic electrocardiographic changes, development of new pathological Q waves on ECG, imaging evidence of myocardial ischemia, or angiographic or autopsy evidence of coronary thrombus).

MINS is defined by at least 1 postoperative cTn concentration that exceeds the 99th percentile upper reference limit of the cTn assay as a result of a presumed ischemic mechanism (ie, supply-demand mismatch or atherothrombosis) in the absence of overt nonischemic causes (eg, pulmonary embolus; Table 1). Such elevations in cTn must be identified within the first 30 days after surgery but nearly always occur within the first 2 postoperative days. Clinical symptoms and electrocardiographic changes are not required to establish a diagnosis of MINS, which includes myocardial infarction and ischemic myocardial injury.

107
Q

A patient in the intensive care unit has ventricular fibrillation two hours after her coronary artery bypass graft procedure. Recommended immediate management does NOT include

a) Atropine
b) Adrenaline
c) 3 stacked shocks
d) Amiodarone

A

b) Adrenaline
-You do not use full dose adrenaline (rather, give smaller doses)
-You do three “stacked shocks”
-You try pacing (rate of 90, DDD) in asystole if pacing wires are available
-If they are already paced and in PEA, you turn off the pacing to “unmask” VF.
-These shocks and attempted pacing are all measures you take before starting CPR, which is a departure from the ACLS norms.
-If you can’t control a shockable rhythm with three stacked shocks, you give amiodarone immediately rather than after three cycles.
-Amiodarone is the only drug in the protocol, which makes it easy to remember. -
-After five minutes of unsuccessful resuscitation the chest should be re-opened. —External CPR is pointless in all of the common causes of arrest in this scenario. Therefore, CPR is something you do while waiting to re-open the chest.
-Non-surgical staff are encouraged to re-open the chest in an emergency

https://derangedphysiology.com/main/required-reading/cardiac-arrest-and-resuscitation/Chapter%20221/cardiac-arrest-following-cardiac-surgery

108
Q

20.2 A woman with atrial fibrillation has no valvular heart disease. According to AHA guidelines, oral anticoagulants are definitely recommended if her CHA2DS2-VASc score is greater than or equal to

A. 1
B. 2
C. 3
D. 4
E. 5

A

C. 3

  • if male CHA2DS2-VASc score ≥2 to be recommended chronic OAC (Grade 1A).
  • if female CHA2DS2-VASc score ≥3 to be recommended chronic OAC (Grade 1A).
  • non-sex risk factor also holds bearing:
  • For patients with CHA2DS2-VASc score of 1 in males and 2 in females based on age 65 to 74 years, we recommend chronic OAC (Grade 1A).

Up to date:

Our approach to deciding whether to prescribe anticoagulant therapy for patients with AF (excluding those with rheumatic mitral stenosis that is severe or clinically significant [mitral valve area ≤1.5 cm2], a bioprosthetic valve [surgical or bioprosthetic] within the first three to six months after implantation, or a mechanical heart valve) is as follows:

*For a CHA2DS2-VASc score ≥2 in males or ≥3 in females, we recommend chronic OAC (Grade 1A).

*For a CHA2DS2-VASc score of 1 in males and 2 in females:
-For patients with CHA2DS2-VASc score of 1 in males and 2 in females based on age 65 to 74 years, we recommend chronic OAC (Grade 1A). Age 65 to 74 years is a stronger risk factor than the other factors conferring one CHA2DS2-VASc score point.
-For patients with other risk factors, the decision to anticoagulate is based upon the specific nonsex risk factor and the burden of AF. For patients with very low burden of AF (eg, AF that is well documented as limited to an isolated episode that may have been due to a reversible cause such as recent surgery, heavy alcohol ingestion, or sleep deprivation), it may be reasonable to forgo chronic OAC and institute close surveillance for recurrent AF, although it may not be possible to reliably estimate AF burden from surveying symptoms or infrequent monitoring. The frequency and duration of AF episodes vary widely over time, and episodes are often asymptomatic.

*For patients with a CHA2DS2-VASc of 0 in males or 1 in females, we suggest against OAC (Grade 2C). Patient values and preferences may impact the decision. For example, a patient who is particularly stroke averse and is not at increased risk for bleeding may reasonably choose anticoagulation, particularly if the patient is a candidate for treatment with a direct oral anticoagulant (DOAC).

2019 AHA/ACC/HRS Focused Update of the 2014 AHA/ACC/HRS Guideline

109
Q

21.1, 21.2 The most common cause of cor pulmonale is

a) Shunt
b) COPD
c) Volume overload
d) PE
e) Kyphoscoliosis

A

Chronic obstructive pulmonary disease (COPD) is the most common cause of cor pulmonale

leads to an increase in RV afterload secondary to changes in pulmonary vascular structure and mechanics, and lung hyperinflation.

Patients with COPD who subsequently develop RV dysfunction have an increased risk of admission to hospital and mortality

110
Q

21.1 A 69-year-old woman has a recent onset of dyspnoea and undergoes a right heart catheterisation, with results displayed below. Her pulmonary capillary wedge pressure is 10 mmHg. The most likely 21.1 diagnosis is

a. Mitral stenosis
b. Mitral regurg
c. Emphysema
d. Pulmonary embolus
e. Aortic stenosis

A

PE

111
Q

20.1 A 15-year-old boy undergoes a cardiac procedure for congenital heart disease. The intrathoracic device is a(n) (chest X-Ray shown)

a) AV repair
b) PV repair
c) ASD closure device
d) Parachute device
e) Right atrial appendage closure
device

A

c) ASD closure device
Amplatzer Device

112
Q

20.2 The 12 lead ECG shown is most consistent with acute total occlusion of the

a) RCA
b) LCx
c) LAD
d) PDA

A

c) LAD

LIFL: Myocardial Ischaemia
https://litfl.com/myocardial-ischaemia-ecg-library/

More Myocardial Ischaemia ECG Examples: Example 1

Subendocardial ischaemia:
The most striking abnormality is the widespread ST depression, seen in leads I, II and V5-6. This is consistent with widespread subendocardial ischaemia.
There is also some subtle ST elevation in V1-2 and aVR with small Q waves in V1-2, suggesting that the cause of the widespread ischaemia is a proximal LAD occlusion.

113
Q

20.2 In the POISE study the use of beta blockers on the day of surgery as a cardio protective strategy in high risk patients has been associated with

a) Increased heart rate
b) Decreased hypotension
c) Increased mortality
d) Increased myocardial infarction

A

c) Increased mortality

Use of perioperative metoprolol was associated with:
* Decreased rate of myocardial infarction
* Decreased rate of revascularisation
* Decreased rate of developing new atrial fibrillation
* INCREASED rate of death
* INCREASED rate of stroke
* INCREASED rate of significant hypotension
INCREASED rate of significant bradycardia

114
Q

22.1 A 78-year-old man is undergoing left heart catheter angiography. A graph displaying pressures in the aorta (Ao) and left ventricle (LV) as well as electrocardiography trace over time is demonstrated below. These pressure recordings are characteristic of

a. MR
b. MS
c. AR
d. AS
e. TR

A

Aortic stenosis

115
Q

22.2 A 56-year-old patient presents with exertional syncope. The most likely diagnosis is
(previously this was a 26yo)

a) HOCM
b) Aortic stenosis
c) Long QT syndrome

A

b) Aortic Stenosis

116
Q

23.1 In cardiac surgery a low-normal central venous pressure and a low blood pressure with a hyperdynamic heart is suggestive of

a. Hypovolaemia
b. Vasoplegia
c. Left ventricular dysfunction

A

b) vasoplegia

117
Q

22.1 This image is an apical four chamber view obtained by transthoracic echocardiography. The artery that supplies the area indicated by the arrow is the

a. RCA
b. LAD
c. CCx

A
118
Q

20.2 Application of a pacemaker magnet to a ventricular implanted pacemaker would be expected to convert the operating mode to

a. DOO
b. VII
c. DDD
d. VVI
e. VOO

A

e. VOO

> Asynchronous mode most often the result of magnet application. In a ventricular PPM, this means VOO

> However, various sources recommend against use of magnet for PPM management due to inconsistent effects on different devices

Equipment in Anaesthesia and Critical Care:

> The use of a magnet as a solution for pacemaker problems, either in theatre or otherwise is not recommended.
The application of a magnet to the pacemaker can have unpredictable results, from causing it to change to a back-up mode such as VOO, to reverting to factory settings, to performing various self-tests, to switching off entirely.

119
Q

22.1 The fourth position of the international pacemaker (NBG) code represents the

A. Pacing
B. Programability
C. Sensing
D. Anti-dyrhythmic functions
E: Inhibition

A

B. Programability ? Multi-site pacing?

120
Q

23.1 According to the Revised Cardiac Risk Index, a 72-year-old male scheduled for a laparoscopic cholecystectomy with a history of hypertension, 20 pack-year history of smoking, type 2 diabetes requiring insulin and a previous stroke has a score of

A. 1
B. 2
C. 3
D. 4
E. 5

A

B. 2
( CVA, Insulin use)

UTD
CHF
Renal impairment
Insulin use
Stroke
Ischaemic heart
Surgically high risk

121
Q

22.2 The drug of choice for the treatment of duct-dependent congenital heart disease is

a) Sildenafil
b) Prostacyclin
c) Carboprost
d) Alprostadil
e) NSAID

A

d) Alprostadil

https://www.rch.org.au/piper/neonatal_medication_guidelines/Alprostadil_(Prostin_VR)%E2%80%93(Prostaglandin_E1)/

Alprostadil (PROSTAGLANDIN E1) is a synthetic prostaglandin used to relax the ductus arteriosus in early post-natal life, where a patent ductus is critical for survival, including Tetralogy of Fallot, pulmonary atresia, pulmonary stenosis, tricuspid atresia and transposition of the great arteries.

Dose
To open a closed ductus arteriosus:
0.1 micrograms/kg/minute (100 nanograms/kg/min). An effect is usually seen within 30-60 minutes. Reduce the dose once an effect is seen or as directed by a Consultant.1

Doses > 0.1 micrograms/kg/minute are rarely more effective and may cause serious adverse effects.3

To maintain patency of ductus arteriosus:
0.01 to 0.02 micrograms/kg/minute (10-20 nanograms/kg/min).1, 2

For persistent pulmonary hypertension of the newborn (PPHN):
0.01 to 0.05 micrograms/kg/minute (10-50 nanograms/kg/min).2

122
Q

23.1 A 75-year-old man has this right heart catheter trace as part of his investigation of dyspnoea. His pulmonary capillary wedge pressure is 24 mmHg. The most likely
diagnosis is:

A. Idiopathic Pulmonary Arterial Hypertension
B. Portopulmonary Syndrome
C. Left Heart Failure
D. Pulmonary Embolism
E. Pulmonary Fibrosis

A

C. Left heart failure causing PulmHTN

Normal PAPs/d is 25/7. This would be classed as severe (55) - (if image is correct)
PAWP >15 means ‘ post-capillary’ cause or combined pre- and post.
This is either group 2 or 5.
A PVR might help differentiate.

All other options (group 1,3,4 and 5) would likely have a isolated ‘pre-capillary’ PAWP of <15

LITFL and blue book 2015 article

123
Q

22.2 When using the ECG to time intra-aortic balloon counterpulsation, balloon inflation should occur at the

a. start of T wave
b. peak of T wave
c. end of T wave
d. end of R wave
e. start of R wave

A

B peak of T wave

Triggering of the IABP is usually set according to the patient’s ECG tracing.
When an R wave is detected the balloon is triggered to automatically start inflating in the middle of the T wave.
Triggering can be impaired if the patient develops an arrhythmia, is paced or has a poor ECG trace.

LITFL

124
Q

23.1 Despite two separate 300 IU/kg doses of heparin, you have failed to attain yourtarget activated clotting time prior to instituting cardiopulmonary bypass. An appropriate option now would be to give

a. More heparin
b. FFP
c. Dalteparin
d. bivalirudin

A

b. FFP

125
Q

22.2 Recirculation is a cannula position complication specific to the use of

a) ECCO2R
b) VV ECMO
c) VA ECMO
d) dialysis
e) AV ECVO

A

b) VV ECMO
VV ECMO Disadvantages
- no cardiac support
- local recirculation though oxygenator at high flows
- reverse gas exchange in lung if FiO2 low
- limited power to create high systemic arterial oxygen tension

126
Q

21.2 A man with atrial fibrillation has no valvular heart disease. According to joint American Heart Association (AHA), American College of Cardiology (ACC) and Heart Rhythm Society (HRS) guidelines, oral anticoagulants are definitely recommended if his CHA2DS2-VASc score is greater than or equal to

a) 1
b) 2
c) 3
d) 4
e) 5

A

b) 2

  • if male CHA2DS2-VASc score ≥2 to be recommended chronic OAC (Grade 1A).
  • if female CHA2DS2-VASc score ≥3 to be recommended chronic OAC (Grade 1A).
  • non-sex risk factor also holds bearing:
  • For patients with CHA2DS2-VASc score of 1 in males and 2 in females based on age 65 to 74 years, we recommend chronic OAC (Grade 1A).

Up to date:

Our approach to deciding whether to prescribe anticoagulant therapy for patients with AF (excluding those with rheumatic mitral stenosis that is severe or clinically significant [mitral valve area ≤1.5 cm2], a bioprosthetic valve [surgical or bioprosthetic] within the first three to six months after implantation, or a mechanical heart valve) is as follows:

*For a CHA2DS2-VASc score ≥2 in males or ≥3 in females, we recommend chronic OAC (Grade 1A).

*For a CHA2DS2-VASc score of 1 in males and 2 in females:
-For patients with CHA2DS2-VASc score of 1 in males and 2 in females based on age 65 to 74 years, we recommend chronic OAC (Grade 1A). Age 65 to 74 years is a stronger risk factor than the other factors conferring one CHA2DS2-VASc score point.
-For patients with other risk factors, the decision to anticoagulate is based upon the specific nonsex risk factor and the burden of AF. For patients with very low burden of AF (eg, AF that is well documented as limited to an isolated episode that may have been due to a reversible cause such as recent surgery, heavy alcohol ingestion, or sleep deprivation), it may be reasonable to forgo chronic OAC and institute close surveillance for recurrent AF, although it may not be possible to reliably estimate AF burden from surveying symptoms or infrequent monitoring. The frequency and duration of AF episodes vary widely over time, and episodes are often asymptomatic.

*For patients with a CHA2DS2-VASc of 0 in males or 1 in females, we suggest against OAC (Grade 2C). Patient values and preferences may impact the decision. For example, a patient who is particularly stroke averse and is not at increased risk for bleeding may reasonably choose anticoagulation, particularly if the patient is a candidate for treatment with a direct oral anticoagulant (DOAC).

2019 AHA/ACC/HRS Focused Update of the 2014 AHA/ACC/HRS Guideline

127
Q

21.1 A third heart sound at the apex may be heard with

a) pulmonary stenosis
b) pulmonary hypertension
c) pericarditis
d) pregnancy

A

d) pregnancy

A third heart sound reflects rapid left ventricular distention along with an increased atrioventricular flow

Heard in Congestive heart failure

Associated with Dilated Cardiomyopathy with dilated ventricles

Less commonly valvular regurgitation and left to right shunts

May be normal physiological finding in patients less than 40yrs old

128
Q

21.1 In cardiac surgery, volatile-based anaesthesia compared to total intravenous anaesthesia

a) Lower 30 day post-op mortality
b) Higher 30 day post-op mortality
c) Lower post-operative MI
d) No difference

A

d) No difference

no observed beneficial effect of sevoflurane on the composite endpoint of prolonged ICU stay, mortality, or both in patients undergoing high-risk cardiac surgery

129
Q

23.1 An adult patient undergoing surgical aortic valve replacement is in ventricular fibrillation after the removal of the aortic cross clamp and requires internal defibrillation. It has been shown it is safe to deliver a charge of up to:

a) 10J
b) 20J
c) 30J
d) 50J
e) 100J

A

B. 50J ?

In internal defibrillation, an initial dose of 20 joules is recommended to avoid burn-like injury to the myocardium. Care should be taken to avoid coronary vessels to prevent vessel damage. Subsequent doses can be increased to a maximum of 40 joules. Sterile internal pads must be used for internal defibrillation and should be readily available during any thoracotomy procedures

Manufacturing manual for internal definition paddles say 50J maximum as higher has been shown to be damaging

https://www.ncbi.nlm.nih.gov/books/NBK499899/

130
Q

21.1 The image below shows a normal central venous pressure (CVP) trace on the left. The CVP trace shown on the right is most consistent with

a. Tricuspid regurg
b. Mitral stenosis
c. Mitral regurg
d. Pericarditis
e. Tamponade

A

a. Tricuspid regurg

Regurgitant CV waves: tricuspid regurgitation

In tricuspid regurgitation, the backflow of blood out of the right ventricle obliterates the normal x descent. The c wave becomes accentuated and fuses with the v wave, as both are the results of right ventricular contraction (and the v wave peak pressure is often the same as the right ventricular peak systolic pressure).

the reality is that they usually fuse completely to produce huge mutant waves, as seen here:

131
Q

21.2 The cardiac axis of this electrocardiogram is

a) -30
b) 0
c) 45
d) -90

A

a) -30

Normal Axis

https://litfl.com/super-axis-man-sam/

LITFL

132
Q

21.2 Findings associated with massive pericardial tamponade include

a) Pulsus paradoxus
b) Electrical alternans
c) Kussmaul sign
d) Pericardial rub

A

a) Pulsus paradoxus electric alternans

Physical findings in Tamponade:
- A number of findings may be present on physical examination, depending upon the type and severity of cardiac tamponade
- None of the findings alone are highly sensitive or specific for the diagnosis.

Beck’s triad
1. Low arterial blood pressure
2. Dilated neck veins
3. Muffled heart sounds
- Are present in only a minority of cases of acute cardiac tamponade.

Physical findings of sinus tachycardia and the absence of frank hypotension
may indicate significant hemodynamic compromise from cardiac tamponade and serve as an indication for immediate pericardiocentesis.

In contrast, Kussmaul sign (the absence of an inspiratory decline in jugular venous pressure) is not usually seen in cardiac tamponade.

Tachycardia and hypotension
- Sinus tachycardia is seen in almost all patients, in an attempt to maintain cardiac output
- Hypotension is somewhat more variably present
- One exception is when the underlying disease is associated with bradycardia, as with a pericardial effusion and subacute cardiac tamponade associated with hypothyroidism.
- Tachycardia also may not be seen in patients with early cardiac tamponade even though they have signs of a hemodynamically significant effusion, such as an elevated jugular venous pressure.

Elevated jugular venous pressure
- The JVP is almost always elevated in cardiac tamponade and may be associated with venous distension in the forehead and scalp.
- CVP wave form:
->x descent is preserved
->y descent is attenuated or absent
(due to the limited or absent late diastolic filling of the ventricle)

Pulsus paradoxus
-defined as: abnormally large decrease in systolic blood pressure (>10 mmHg) on inspiration
- common finding in moderate to severe cardiac tamponade and is the direct consequence of ventricular interdependence.
- not all patients with cardiac tamponade have pulsus paradoxus (eg, those with chronic hypertension leading to elevated ventricular diastolic pressures or those with a co-existent atrial septal defect).

Pericardial rub — A pericardial rub may be heard in patients with cardiac tamponade due to inflammatory pericarditis.

Electrocardiography
- ECG in cardiac tamponade typically shows sinus tachycardia and may also show low voltage.
- If pericarditis is present, the ECG findings typical of that disorder are also seen.
- Electrical alternans is characterized by beat-to-beat alterations in the QRS complex and, in some cases, other electrocardiographic waves that reflect the swinging of the heart in the pericardial fluid.
- Electrical alternans is relatively specific but not very sensitive for cardiac tamponade; rarely, this phenomenon is seen with very large pericardial effusions alone.

Diagnosis:
Clinical diagnosis is usually suspected based on the history and physical examination findings, which may include:
●Chest pain
●Syncope or presyncope
●Dyspnea and tachypnea
●Hypotension
●Tachycardia
●Peripheral edema
●Elevated jugular venous pressure
●Pulsus paradoxus

Presence of a pericardial effusion on echocardiography with evidence of cardiac chamber collapse, flow variation, or dilation of the inferior vena cava is consistent with, and highly suggestive of, cardiac tamponade.

However, the diagnosis of cardiac tamponade can only be confirmed by the hemodynamic and clinical response to pericardial fluid drainage.

133
Q

20.2 A patient is in sinus rhythm at a heart rate of 60 /minute and a QT interval of 420 msec. The corrected QT interval is

a) 360 ms
b) 380 ms
c) 420 ms
d) 460 ms
e) 480 ms

A

c) 420 ms
i.e. RR interval = 60 / HR; therefore RR interval is 60/60 = 1
420 / square root of 1 is 420ms

From LITFL:
The corrected QT interval (QTc) estimates the QT interval at a standard heart rate of 60 bpm
QT:
Time from the start of the Q wave to the end of the T wave
Represents time taken for ventricular depolarisation and repolarisation, effectively the period of ventricular systole from ventricular isovolumetric contraction to isovolumetric relaxation
The QT interval is inversely proportional to heart rate:
The QT interval shortens at faster heart rates
The QT interval lengthens at slower heart rates
An abnormally prolonged QT is associated with an increased risk of ventricular arrhythmias, especially Torsades de Pointes

Corrected QT interval (QTc)
The corrected QT interval (QTc) estimates the QT interval at a standard heart rate of 60 bpm
This allows comparison of QT values over time at different heart rates and improves detection of patients at increased risk of arrhythmias
Bazett formula: QTC = QT / √ RR

Note: The RR interval is given in seconds (RR interval = 60 / heart rate).

134
Q

22.1 To allow cardiopulmonary bypass in a patient with heparin resistance, fresh frozen plasma may be administered in order to increase the level of

A

ATIII

135
Q

22.2 A patient with acute right heart failure secondary to acute myocardial infarction is likely to have a/an

a. Increased PA pulsatility index
b. Increased tricuspid annular plane systolic excursion
c. Decreased PAP
d. Raised JVP
e. Decreased PA pulsatility index

A

PAPi goes down in acute RVF
- actually designed to prognosticate in acute MI

E is the answer

https://www.ahajournals.org/doi/10.1161/CIRCHEARTFAILURE.121.009085#:~:text=The%20pulmonary%20artery%20pulsatility%20index,left%20ventricular%20assist%20device%20implantation.

136
Q

22.2 Drug classes demonstrated to reduce mortality in chronic heart failure with reduced ejection fraction include all of the following EXCEPT

A. ACE inhibitors
B. Beta blockers
C. Angiotensin receptor blockers
D. Spironolactone
E. Digoxin

A

Digoxin

137
Q

21.2 High-risk transthoracic echocardiogram findings associated with aortic dissection include all of the following EXCEPT

a) RWMA
b) Pericardial effusion
c) Dilated aortic root
d) Aortic regurgitation
e) LV hypertrophy

A

E) LV hypertrophy

ECHO FINDINGS

intimal flap

TYPING (type A):
aortic regurgitation (acute dilatation of the aortic root, aortic leaflet prolapse, dissection flap prolapse, pre-existing disease, e.g. bicuspid valve)

pericardial effusion and/or tamponade

regional wall motion abnormality heralding coronary artery occlusion

DOPPLER
identifies true and false lumen

detect aortic branch occlusion/ dissection (absent flow)

Source LITFL

138
Q

22.2 You are performing a focused cardiac ultrasound in the postanaesthesia care unit on a patient who is hypotensive for unclear reasons. His heart rate is 100 beats/min. The left ventricular velocity time integral is 10 cm. The left ventricular outflow area is 3 cm2. The left ventricular ejection fraction is 25%. The right ventricular systolic pressure is 40 mmHg. The inferior vena cava diameter is 20 mm. The estimated cardiac output is

a. 1L/min
b. 2L/min
c. 3L/min
d. 4L/min
e. 5L/min

A

C 3L/min

LVOT area x VTI = SV
3cm2 x 10cm = 30ml

SV x HR = CO
30 x 100 = 3000

139
Q

22.1 A patient requires a peripherally inserted central venous catheter. Electrocardiographic (ECG)-aided tip localisation is used to site the tip of the catheter. The initial ECG from the catheter is shown.

The ECG when the catheter is placed appropriately will be

A

Maximum P-wave corresponds to placement in the Cavo-atrial Junction (CAJ)

Benefits of CAJ placement
- furthest distance from “high risk” areas
- Largest vein diameter
- highest blood flow capacity
- minimal risk for catheter migration and looping

Too short placement:
Increased risk of:
- DVT
- Phlebitis

Too long placement:
Increased risk of:
- arrhythmias
- tricuspid valve dysfunction
atrial dysfunction

ECG and corresponding anatomy:

  • Normal P-wave = upper vasculature prior to CAJ
  • Max P-wav= CAJ
  • Initial negative P-wave deflection = Right Atrium
  • Biphasic P-wave = Right Atrium
  • Inverted P-wave = Right Ventricle
140
Q

20.2 The maximum warm ischaemic time (in minutes) acceptable for procuring the lungs following donation after cardiac death is

a. 30
b. 60
c. 90
d. 120
e. 240

A

c. 90

Warm ischaemia time:
- Time from treatment withdrawal to the start of cold perfusion of the donated organs
- Significance is the impact on graft function
- Most important phase of WIT begins when the systolic BP is < 60mmHg
- This includes the waiting period from the absence of circulation to the declaration of death and the time before initiating the flow of cold perfusate through the cannula

Maximum WARM Ischaemia time
- Heart 30 mins
- Liver 30 mins
- Pancreas 30 mins
- Kidney 60 mins
- Lungs 90 mins

Maximum COLD Ischaemia time:
- Heart = 4 hrs
- Lungs = 6-8hrs
- Liver/Pancreas = 12hrs (DBD)/6 hrs (DCD)
- Kidneys = 18hrs (DBD)/ 12 hrs (DCD)

141
Q

20.1 A 75-year-old man has this right heart catheter trace as part of his investigation of dyspnoea. His pulmonary capillary wedge pressure is 24mmHg. The most likely diagnosis is (pressure trace shown)
Right heart cath: PA pressure 75/26, CWP24

a) Pulmonary Arterial Hypertension
b) LV failure
c) Pulmonary embolus
d) Emphysema
E) aortic stenosis

A

Left heart failure (pulmonary htn with raised Left sided pressure (PCWP 24mmHg)

PCWP >15mmHg.
Therefore post-capillary or combined pre-and post-capillary PH = Left heart failure (clinical group 2)
Other Options are clinical groups 1,3,4 of PH and therefore not correct (as characteristic is PAWP<15mmHg).

WHO pulmonary Htn classification
1: Primary arterial Hypertension
2: 2ry to left heart disease
3: 2ry to lung disease/hyypoxia
4: Chronic Thromboembolic disease
5: Unclear/multifactorial

142
Q

22.2 When using cardioversion to revert a patient in atrial fibrillation to sinus rhythm, the direct current shock is synchronised with the ECG to coincide with the

a) Start of R wave
b) Start of Q wave
c) Middle of T wave
d) Peak of R wave

A

d) Peak of R wave

The appropriate energy level is then selected, and the discharge/shock button is pressed and held. The defibrillator does not release the shock immediately. Instead, it waits for the next R-wave to appear and delivers the shock at the time of the R-wave. This allows the shock to be provided safely away from the T wave, avoiding the R-on-T phenomenon.

143
Q

21.1 The following ECG is consistent with

a) Atrial fibrillation
b) Lown-Ganong-Levine (LGL) Syndrome
c) Previous Myocardial Infarction
d) Wolf-Parkinson White Type B
d) Left Ventricular Hypertrophy

A

D) WPW B

Sinus rhythm with very short PR interval (< 120 ms)
Broad QRS complexes with a slurred upstroke to the QRS complexes — the delta wave
Dominant S wave in V1 indicates a right-sided accessory path — sometimes referred to as “Type B” WPW
Tall R waves and inverted T waves in the inferior leads and V4-6 mimic the appearance of left ventricular hypertrophy (LVH) — again, this is due to WPW and does not indicate underlying LVH

144
Q

A 72-year-old man with a murmur has a left heart catheter. Shown are the simultaneous waveforms in the aorta and left ventricle. The most likely diagnosis is

A. Aortic regurgitation
B. Aortic Stenosis
C. HOCM
D. Mitral regurg
E. Normal

A

Answer: A Aortic regurgitation
- lack of dichrotic notch, high pulse pressure

https://www.ahajournals.org/doi/full/10.1161/CIRCULATIONAHA.111.060319#d1e195

145
Q

22.1 You are anaesthetising a patient for implantation of an automated implantable cardioverter defibrillator. The patient is a 48-year-old with dilated cardiomyopathy and pulmonary hypertension.

The preoperative echocardiogram report states that the estimated pulmonary artery systolic pressure is 55 mmHg, and that there is mild right ventricular systolic dysfunction. To avoid
worsening right ventricular function during induction, it would be best to consider using

a. Milrinone
b. Dopamine
c. Dobutamine
d. Adrenaline

A

c. Dobutamine

In 2017 a similar questions was asked and an option for metaraminol was given, metaraminol could be a better answer as it will increase systemic pressure and reduce heart rate maintaining RCA perfusion at induction. Dobutamine and Milrinone can cause systemic vasodilation leading to reduction in systemic blood pressure and RCA perfusion pressure, Both adrenaline and Dopamine do not cause pulmonary vasodilation and can lead to tachyarhythmias

Pulmonary hypertension and its management in patients undergoing non-cardiac surgery
https://associationofanaesthetists-publications.onlinelibrary.wiley.com/doi/10.1111/anae.12831

Vasoconstrictors, inotropes and inodilators

Maintaining the gradient between aorta and right ventricle is achieved by using sympathomimetic and non-sympathomimetic vasopressors. Noradrenaline and vasopressin improve perfusion of the right coronary artery, reduce the pulmonary/systemic vascular resistance ratio, enhance right ventricular performance and marginally improve cardiac output

However, the evidence of their impact on mortality related to right heart failure is weak. Inotropes that enhance right ventricular performance, such as adrenaline, dobutamine and levosimendan are effective in treating right-sided heart failure.

The use of inotropes has a modest impact in reducing the overall mortality related to PH, and their wide availability and ease of administration make this group of drugs very attractive for use in the peri-operative setting.

Inodilators, such as the phosphodiesterase-3 inhibitors milrinone and enoximone, have been shown to be beneficial when compared with conventional inotropic support only. It appears that the influence of phosphodiesterase-3 inhibitors on reducing pulmonary vascular resistance is more pronounced than the reduction in systemic vascular resistance. However, reduction in systemic vascular resistance can compromise right coronary artery blood flow in patients with severe PH and therefore they should be administered cautiously.

Treatment of pulmonary hypertensive crisis:

General principles
- Avoid hypoxic pulmonary vasoconstriction
- Avoid hypercarbia, acidosis and hypothermia
- Avoid high airway pressures
- Optimise right ventricular preload
- Reduce right ventricular afterload
- Maintain coronary blood flow
- Maintain sinus rhythm
- Maintain arterial blood pressure and cardiac output

Vasopressors– noradrenaline; vasopressin

Inotropes– adrenaline; dobutamine

Inodilators– milrinone; enoximone

Intravenous vasodilators (caution if low systolic blood pressure)
- Milrinone (25–50 μg.kg−1 bolus, followed by 0.5–0.75 μg.kg−1.min−1 continuous infusion)
- Prostacyclin (4–10 ng.kg−1.min−1 continuous infusion)
- Iloprost (1–3 ng.kg−1.min−1 continuous infusion)
- Sildenafil (10 mg bolus three times a day)

Selective pulmonary vasodilation
- Iloprost (5–10 μg diluted in 10 ml saline, nebulised over 10 min, repeated every 2–4 h)
- Prostacyclin (25–50 μg diluted in 50 ml saline, nebulised over 15 min, repeated every hour)
- Nitric oxide (5–40 ppm continuously)

146
Q

23.1 Of the following drugs, the LEAST likely to cause pulmonary vasodilation when used at low doses in patients with chronic pulmonary hypertension is

a) Vasopressin
b) Dobutamine
c) Dopamine
d) Milrinone

A

Dopamine

  • least likely to cause pulmonary vasodilation (all the others do to my knowledge)
  • From UP TO DATE:
    > At low doses of 1 to 3 mcg/kg per min, dopamine acts primarily on dopamine-1 receptors to dilate the renal and mesenteric artery beds
    > At 3 to 10 mcg/kg per min (and perhaps also at lower doses), dopamine also stimulates beta-1 adrenergic receptors and increases cardiac output, predominantly by increasing stroke volume with variable effects on heart rate.
    > At medium-to-high doses, dopamine also stimulates alpha-adrenergic receptors, although a small study suggested that renal arterial vasodilation and improvement in cardiac output may persist as the dopamine dose is titrated up to 10 mcg/kg per min
    *clinically, the haemodynamic effects of dopamine demonstrate individual variability

Dobutamine (inodilator):
- selective β1-agonist that increases cardiac contractility and reduces pulmonary vascular and systemic vascular resistances

Vasopressin:
- vasopressin may have pulmonary vasodilatory effects in addition to a systemic vasoconstrictive effect

Milrinone (inodilator):
- the phosphodiesterase-3 inhibitors, milrinone and enxoimone, have positive inotropic effects combined with the capacity to reduce RV afterload (‘inodilators’) without significant chronotropic effect, but they can be associated with significant systemic hypotension

https://pubs.asahq.org/anesthesiology/article/121/5/914/13855/VasopressinThe-Perioperative-Gift-that-Keeps-on

147
Q

21.1 High-risk transthoracic echocardiogram findings associated with aortic dissection include all of the following EXCEPT

a) pericardial effusion
b) dilated Ao root
c) RV dilatation
d) RWMA
e) AR

A

EXCEPT C) RV dilation

Echo findings in Aortic Dissection:
1. Intimal flap

  1. Type A dissection:
    - Aortic regurgitation
    -Acute dilation of aortic root
    -Aortic leaflet prolapse
    -Dissection flap prolapse
    -Pre-existing disease
    -Pericardial Effusion/Tamponade
    -RWMA
  2. Colour flow doppler
    -identifies true and false lumen
    -aortic branch occlusion/dissection
148
Q

21.1 An ECG abnormality which is NOT usually associated with severe anorexia nervosa is

a) QT prolongation
b) TWI
c) ST depression
d) prolonged PR interval
e) sinus tachycardia

A

e) sinus tachycardia

https://academic.oup.com/bjaed/article/9/2/61/299563

Typically anorexic patients are hypotensive and bradycardic.
Bradycardia reflects the decrease in basal metabolic rate that arises as an adaptive response to starvation
Electrocardiographic abnormalities are common and may be found in >80% of strict dieters. These include:
1. atrioventricular block
2. ST depression
3. T wave inversion
4. QT prolongation.

149
Q

23.1 The tip of an ideally-placed intra-aortic balloon catheter should lie in the

A. Distal to aortic root
B. Distal to left subclavian artery
C. Distal to left carotid
D. Distal to renal veins.

A

B. distal to LSCA

The appropriate performance of the IABP is dependent on proper position. Ideally, the tip of the balloon should be positioned 2–3 cm distal to the origin of the left subclavian artery (LSCA)

https://academic.oup.com/bja/article/110/2/316/228037

150
Q

21.1 The following is an image from a focussed cardiac ultrasound in a patient with dyspnoea presenting for thoracic surgery. The diagnosis is

a) Pericardial effusion
b) Tamponade
c) Pleural effusion
d) Loculated lung abscess

A

c) Pleural effusion

Source: BJA Ultrasound in critical care

Thoracic US revealing a large pleural effusion (E) that has displaced the lung. The diaphragm (D) and liver (L) are visualized. The depth from the skin to the fluid has been measured. Insertion of the needle at this site is not advised; given the proximity to the liver, a more superior approach should be marked.

151
Q

21.2 A 74-year old man in the post-anaesthesia care unit complains of chest pain. An electrocardiogram (ECG) is performed. The occluded coronary artery is the

a) RCA
b) LCx
c) LAD
d) PDA

A

RCA (Inferior STEMI)
- 80% RCA
- 18% LCx
- 2% rare wrap around LAD

ST elevation in lead III > lead II
Presence of reciprocal ST depression in lead I
Signs of right ventricular infarction: STE in V1 and V4R

Source LITFL

152
Q

20.1 Of the following, the maternal cardiac condition that represents the highest risk of mortality associated with pregnancy is

a. Bicuspid aortic valve with significantly dilated aortic root
b. Fontan circulation
c. HOCM
d. PDA

A

A - Modified WHO class 4

B - Modified WHO class 3 (4 if ANY complication)
C - Modified WHO class 2-3 (if severe AS - 4)
D - Modified WHO class 1

Class 4 = 40-100% risk of event

Source
https://academic.oup.com/eurheartj/article/39/34/3165/5078465

Table:
https://academic.oup.com/view-large/186437995

153
Q

23.1 This 12 lead ECG shows

A. Complete heart block
B. Mobitz I
C. Mobitz II
D. LPFB + RBBB
E. LAFB + RBBB

A

E
LAFB:
> Left axis deviation (usually -45 to -90 degrees)
> qR complexes in leads I, aVL
> rS complexes in leads II, III, aVF
> Prolonged R wave peak time in aVL > 45ms

LPFB:
> Right axis deviation (RAD) (> +90 degrees)
> rS complexes in leads I and aVL
> qR complexes in leads II, III and aVF
> Prolonged R wave peak time in aVF

154
Q

22.1 You are about to anaesthetise a 25-year-old man for an open appendicectomy. He has a history of tricuspid atresia for which he has had a Fontan procedure. An important goal in managing his ventilation under anaesthesia is to ensure

a. Long I time, low pressures
b. Long I time, PEEP
c. Long E time
d. Spontaneous ventilation

A

c. Long E time

Spont vent not appropriate for this surgery as will require RSI so spont vent can’t be ensured

BJA: fontan circulation:
For relatively short procedures, Fontan patients are probably better off breathing spontaneously, as long as severe hypercarbia is avoided. For major surgery, or when prolonged anaesthesia is required, control of ventilation and active prevention of atelectasis is usually advisable. Potential disadvantages of mechanical ventilation in Fontan patients relate to the inevitable increase in mean intrathoracic pressure. This causes decreased venous return, decreased pulmonary blood flow, and hence, decreased cardiac output. Low respiratory rates, short inspiratory times, low PEEP, and tidal volumes of 5–6 ml kg−1 usually allow adequate pulmonary blood flow, normocarbia, and a low PVR. Hyperventilation tends to impair pulmonary blood flow, despite the induced respiratory alkalosis, because of the increased mean intrathoracic pressure.

https://academic.oup.com/bjaed/article/8/1/26/277637

155
Q

20.1 A 55-year-old man is found to be in atrial fibrillation. He has no previous medical history. Physical examination, blood pressure and fasting blood glucose are normal. Appropriate long-term management is

A. Aspirin
B. Dabigatran
C. No anticoagulation
D. Warfarin
E. Rivaroxaban

A

C. No Anticoagulation

  • if male CHA2DS2-VASc score ≥2 to be recommended chronic OAC (Grade 1A).
  • if female CHA2DS2-VASc score ≥3 to be recommended chronic OAC (Grade 1A).
  • non-sex risk factor also holds bearing:
  • For patients with CHA2DS2-VASc score of 1 in males and 2 in females based on age 65 to 74 years, we recommend chronic OAC (Grade 1A).

Up to date:

Our approach to deciding whether to prescribe anticoagulant therapy for patients with AF (excluding those with rheumatic mitral stenosis that is severe or clinically significant [mitral valve area ≤1.5 cm2], a bioprosthetic valve [surgical or bioprosthetic] within the first three to six months after implantation, or a mechanical heart valve) is as follows:

*For a CHA2DS2-VASc score ≥2 in males or ≥3 in females, we recommend chronic OAC (Grade 1A).

*For a CHA2DS2-VASc score of 1 in males and 2 in females:
-For patients with CHA2DS2-VASc score of 1 in males and 2 in females based on age 65 to 74 years, we recommend chronic OAC (Grade 1A). Age 65 to 74 years is a stronger risk factor than the other factors conferring one CHA2DS2-VASc score point.
-For patients with other risk factors, the decision to anticoagulate is based upon the specific nonsex risk factor and the burden of AF. For patients with very low burden of AF (eg, AF that is well documented as limited to an isolated episode that may have been due to a reversible cause such as recent surgery, heavy alcohol ingestion, or sleep deprivation), it may be reasonable to forgo chronic OAC and institute close surveillance for recurrent AF, although it may not be possible to reliably estimate AF burden from surveying symptoms or infrequent monitoring. The frequency and duration of AF episodes vary widely over time, and episodes are often asymptomatic.

*For patients with a CHA2DS2-VASc of 0 in males or 1 in females, we suggest against OAC (Grade 2C). Patient values and preferences may impact the decision. For example, a patient who is particularly stroke averse and is not at increased risk for bleeding may reasonably choose anticoagulation, particularly if the patient is a candidate for treatment with a direct oral anticoagulant (DOAC).

2019 AHA/ACC/HRS Focused Update of the 2014 AHA/ACC/HRS Guideline

156
Q

21.2 A drug which is likely to slow the heart rate in a patient with a heart transplant is

a. Phenylephrine
b. Digoxin
c. Metaraminol
d. Adenosine

A

Adenosine (effect is exagerated)

157
Q

22.1 The radial artery pressure trace shown below is from a patient who has an intra-aortic balloon pump in situ. The device has been switched to 1:2 augmentation to assess the timing. The trace shows an augmented beat followed by a nonaugmented beat. With respect to the augmentation, the trace shows

a. Early deflation
b. Late deflation
c. Late inflation
d. Early inflation
e. No change

A

c. Late inflation

158
Q

21.1 The apical four–chamber view of a transthoracic echocardiogram below shows all EXCEPT
a) Left pulmonary vein
b) Pulmonary Trunk
c) Right Atrium
d) Aortic valve

A

b) pulmonary trunk

159
Q

22.2 The image below shows a normal central venous pressure (CVP) trace on the left. The CVP trace shown on the right is most consistent with

(actual image on exam! found on deranged physiology)
a. Tricuspid regurg
b. Constrictive pericarditis
c. Restrictive cardiomyopathy
d. Cardiac tamponade
e. Complete heart block

A

d. Cardiac tamponade

Deranged physiology

In summary
The CVP is raised
All CVP waveform components are elevated
a and v waves are tall
x descent is steep
y descent is (usually) absent

160
Q

20.1
a. Left anterior hemiblock
b. Left posterior hemiblock
c. RBBB
d. LBBB

A

First degree heart block + Left Anterior hemiblock

ECG criteria for LAFB/LAHB

Left axis deviation (usually -45 to -90 degrees)
qR complexes in leads I, aVL
rS complexes in leads II, III, aVF
Prolonged R wave peak time in aVL > 45ms
161
Q

23.1 A 35-year-old woman is brought to the emergency department following a suspected amitriptyline overdose. She has a Glasgow Coma Scale score of 6 and her blood pressure is 90/46 mmHg. Her electrocardiogram is most likely to show

A. AF
B. CHB
C. Sinus tachy with prolonged QRS
D. Sinus brady with prolonged QRS
E. VT

A

c. sinus tachy with prolonged QRS

162
Q

The most likely diagnosis for the following electrocardiograph is

a) VF
b) AF w bundle branch block
c) SVT w BBB
d) VT
e) Sinus w BBB

A

d) VT

https://litfl.com/ventricular-tachycardia-monomorphic-ecg-library/

163
Q

Pulmonary hypertension is defined as a mean pulmonary arterial pressure greater than
a) 15mmHg
b) 20mmHg
c) 25mmHg
d) 30mmHg

A

b) 20mmHg

164
Q

According to the ATACAS trial, the continuation of low-dose aspirin prior to cardiac surgery is associated, in the postoperative period, with

a) No increased risk of bleeding
b) Decreased risk of MI
c) Increased risk of Thrombotic events
d) Increased risk of seizures

A

a) No increased risk of bleeding

There is no evidence that pre-operative aspirin administration resulted in a lower risk of death or thrombotic complications, or a higher risk of haemorrhage.

The study aim (and title) was to compare stopping vs continuing aspirin, however the design insisted on all patients stopping aspirin and then being given a single dose of aspirin or placebo prior to surgery (and presumably all patients were given aspirin after surgery) – this method hasn’t really investigated the theory

TheBottomLine.org.uk

165
Q

Synchronised direct current cardioversion is NOT indicated when the arrhythmia is

a) AF
b) Flutter with rate <100
c) Multifocal atrial tachy
d) SVT with
e) Conscious torsades

A

C- Multifocal Atrial Tachycardia

Cardioversion is contraindicated in MAT. Due to the multiple atrial foci, direct current (DC) cardioversion is not effective in restoring normal sinus rhythm and can precipitate more dangerous arrhythmias.
- https://emedicine.medscape.com/article/155825-overview#a10

DCCV is indicated for
1. Any haemodynamically unstable narrow or wide QRS complex tachycardia
2. AF <48hrs
3. AF >48hrs with adequate anticoag/TOE to exclude thrombus
4. SVTs and monomorphic TVs not responding to trial of IV medical therapy

DCCV is CONTRAindicated in:
a. Digitalis toxicity and associated tachycardia
b. AF >48hrs without adequate anticoagulation/TOE
-BJAEducation 2017
https://academic.oup.com/bjaed/article/17/5/166/2669966

166
Q

In a cardiac transplant recipient, hypotension due to general anaesthesia is least likely to respond to

a) noradrenaline
b) Ephedrine
c) adrenaline
d) Atropine

A

d) Atropine

Blue book 2019

167
Q

The Myocardial Injury after Non Cardiac Surgery study showed elevated troponin in the first three post-operative days was strongly associated with

a) 30 day mortality
b) 30 day MI

A

New question.

a) 30 day mortality

Postoperative myocardial injury was associated with an increased risk of death. Twenty-seven of the 315 patients (8.6%; 95% CI, 6.0–12.2%) with myocardial injury died within 30 days compared with 29 of the 1312 patients (2.2%; 95% CI, 1.5–3.2%) with normal troponin I levels (P<0.01)

Reference: Myocardial Injury After Noncardiac Surgery and its Association With Short-Term Mortality (Circulation 2013)

168
Q

A patient with a perioperative troponin rise above the upper limit of normal, chest pain, left ventricular anterior regional wall motion abnormality, and atheroma with a partially occluding thrombus of the left anterior descending coronary artery has had a/an

A) Type 1 MI
B) Type 2 MI
C) NSTEMI
D) MINS

A

A) Type 1 MI

Clinical classification based on the assumed proximate cause of the MI:
- Type 1
○ MI caused by atherothrombotic coronary artery disease
○ And usually precipitated by atherosclerotic plaque disruption ( rupture or erosion)
- Type 2
○ MI consequent to a mismatch between oxygen supply and demand
○ Multiple potentional mechanisms:
§ Coronary dissection
§ Vasospasm
§ Emboli
§ Microvascular dysfunction
§ Increases in demand without underlying coronary artery disease
- Type 3
○ Patient with typical presentation of MI (ECG changes or VF) with unexpected death before blood samples for biomarkers could be drawn
- Type 4a
○ MI associated with Percutaneous Coronary intervion (PCI)
- Type 4b
○ Subcategory of PCI related MI related to stent/scaffold thrombosis
- Type 5
CABG related MI

169
Q

A patient has received high dose hydroxycobalamin for refractory vasoplegia post cardiac
surgery. Observed effects include all of the following EXCEPT

a) leukopenia
b) red urine
c) falsly low SpO2
d) thrombocytosis

A

c) falsly low SpO2

Effects of hihg dose hydroxycobalamin:
- red urine
- thrombocytosis
- leukopenia

170
Q

An inverted u wave is an electrocardiographic sign of

a) Hypokalaemia
b) Raised ICP
c) Digoxin treatment
d) Myocardial ischaemia

A

D> Myocardial ischaemia

An inverted U wave may represent myocardial ischemia (and especially appears to have a high positive predictive accuracy for left anterior descending coronary artery disease[7] ) or left ventricular volume overload.
^Wikipedia
——–
U-wave inversion is abnormal (in leads with upright T waves)
A negative U wave is highly specific for the presence of heart disease
Common causes of inverted U waves

Coronary artery disease
Hypertension
Valvular heart disease
Congenital heart disease
Cardiomyopathy
Hyperthyroidism
In patients presenting with chest pain, inverted U waves:

Are a very specific sign of myocardial ischaemia
May be the earliest marker of unstable angina and evolving myocardial infarction
Have been shown to predict a ≥ 75% stenosis of the LAD / LMCA and the presence of left ventricular dysfunction
^LITFL: https://litfl.com/u-wave-ecg-library/

171
Q

The use of direct oral anticoagulants [DOAC] in atrial fibrillation is contraindicated in the
presence of

a) Bioprosthetic Heart Valve
b) Mitral Regurgitation
c) mild hepatorenal impairment
d) Mitral Stenosis, moderate to severe

A

D) Mitral Stenosis (Rheumatic, moderate to severe)

DOAC use is contraindicated in certain clinical conditions, notably, in patients who have a mechanical heart valve and those with rheumatic mitral stenosis. Moderate to severe renal impairment or significant hepatic disease is also a contraindication to DOAC treatment

Bioprosthetic valves are less thrombogenic thus DOAC use is acceptable. https://www.ahajournals.org/doi/epdf/10.1161/JAHA.120.017559

172
Q

When using the ECG to time intra-aortic balloon counterpulsation, balloon deflation should occur at the

a. start of T wave
b. peak of T wave
c. end of T wave
d. end of R wave
e. peak of R wave

A

e. peak of R wave

https://derangedphysiology.com/main/required-reading/cardiothoracic-intensive-care/Chapter%20634/normal-iabp-waveform

https://litfl.com/intra-aortic-balloon-pump-trouble-shooting/

173
Q

A 58-year-old man with ischaemic cardiomyopathy is undergoing a ventricular tachycardia ablation procedure in the catheter laboratory. Partway through the procedure his systolic blood pressure abruptly falls from 110 mmHg to 50 mmHg. The most likely cause for his hypotension is

a) Tamponade
b) RV failure
c) Arrhythmia
d) Anaphylaxis
e) Oesophageal aortic fistula

A

a) Tamponade

Cardiac tamponade occurs ~1%. Can usually be managed with reversal of anticoagulation and percutaneous drainage.

Vascular complications most common followed by tamponade.

https://www.ahajournals.org/doi/10.1161/circep.113.000768

https://academic.oup.com/bjaed/article/12/5/230/289246#3659733

174
Q

The coronary artery most likely occluded in this ECG of an acute ST-elevation myocardial
infarction is the

a) RCA
b) L Cx
c) LAD
d) Left Main CA
e) Posterior Descending CA

A

c) LAD

175
Q

The antiemetic least likely to precipitate an arrhythmia in a patient with this ECG is

a) Droperidol
b) Metoclopramide
c) Promethazine
d) Dexamethasone
e) Ondansetron

A

d) Dexamethasone
The ECG shows LONG QT

https://litfl.com/qt-interval-ecg-library/